Human Physiology

Réussis tes devoirs et examens dès maintenant avec Quizwiz!

2Q30You are walking to class, pondering the intricacies of physiology, when you trip over an uneven place in the sidewalk, and fall. Unhurt but embarrassed and angry, you jump up and glance around to see if anyone is watching. From your knowledge of neuroanatomy and function, explain how the following areas of the brain might be involved in this scenario: A. cerebrum B. cerebellum C. limbic system

A. cerebrum = realize that you just fell (thought), so you jump up quickly (action) B. cerebellum = you glance around to see who's watching C. limbic system = you get embarrassed and angry

2-29Energy is defined as A) the light and heat from the sun. B) the capacity to do work. C) using glucose to synthesize ATP. D) doing tasks that make you tired.

B

2Q23The bones within the middle ear connect the A) cochlea to the tympanic membrane. B) tympanic membrane to the oval window. C) oval window to the round window. D) cochlea to the oval window. E) tympanic membrane to the round window.

B

2Q24Neurotransmitters are released from the A) synapse. B) axon terminals. C) axon hillock. D) dendrites. E) collaterals.

B

2Q34Match the description with the drug type: Blocks secretion of pancreatic enzymes A) sympathetic antagonist B) parasympathetic antagonist

B

2Q35Match the description with the drug type: Blocks urination A) sympathetic antagonist B) parasympathetic antagonist

B

2Q7The larger the receptive field, the A) fewer primary sensory neurons are involved. B) more primary sensory neurons synapse on a secondary sensory neuron. C) larger the area of the somatosensory cortex in the brain that perceives the sensation. D) stronger the stimulus intensity required to activate a sensory receptor.

B

3-4Which of the following is NOT a method of hormone action? A) control of electrical signaling pathways B) All are methods of hormone action. C) control of enzymatic reaction rates D) control of ion or molecule transport across cell membranes E) control of gene expression and protein synthesis

B

3-5The link between a first messenger and a second messenger in a cell that responds to peptide hormones is usually A) adenylyl cyclase. B) a G protein. C) cAMP. D) cAMP. E) calcium.

B

3Q15A motor unit consists of A) one axon terminal branch and the myofibrils it controls. B) one neuron and all the skeletal muscle fibers it controls. C) one skeletal muscle fiber and all the neurons that control that fiber. D) one skeletal muscle fiber and the neuron that controls that fiber.

B

3Q1621. When blood pressure receptors sense a loss of blood pressure, they ________ their firing rate. A) increase B) decrease

B

3Q23If cardiac output increases and resistance in arterioles does NOT change, what happens to arterial blood pressure? A) is unchanged B) increases C) decreases

B

3Q27Perfusion is A) delivery of oxygen to cells. B) blood flow through an organ. C) the connection between capillaries and other vessels. D) movement of blood through a shunt. E) the driving force behind blood flow.

B

3Q9Air moves into the lungs because A) contraction of the diaphragm decreases the volume of the pleural cavity. B) the gas pressure in the lungs is less than outside pressure. C) the volume of the lungs decreases with inspiration. D) the thorax is muscular. E) All of the answers are correct.

B

4-13The Nernst equation predicts A) intracellular ion concentrations B) the membrane potential resulting from permeability to a single ion. C) the threshold membrane potential. D) the membrane potential resulting from all permeable ions. E) extracellular ion concentrations

B

5-20The neural pathway from the spinal cord to the target tissue has two neurons, the preganglionic neuron and the postganglionic neuron. A) true only for the parasympathetic division B) true for both divisions C) true only for the sympathetic division

B

5-6Which of the following has its cell body in the ganglion? A) somatic motor neuron B) postganglionic neuron C) preganglionic neuron D) preganglionic neuron, postganglionic neuron, and somatic motor neuron E) preganglionic neuron and postganglionic neuron

B

5-7It is important during stress or emergencies (fight-or-flight). A) true for both divisions B) true only for the sympathetic division C) true only for the parasympathetic division

B

5-8Inside the ganglia are interneurons, which modulate messages. A) true only for the parasympathetic division B) true for both divisions C) true only for the sympathetic division

B

6-10Which class of movement can be considered a combination of the other two? A) voluntary B) rhythmic C) reflex

B

6-9Identify the FALSE statement. A) Adipose tissue is controlled by autonomic efferents. B) All reflexes require input from the brain. C) Some reflexes are genetically determined. D) Muscle spindles are stretch receptors. E) Proprioceptors detect limb position and movement.

B

7-18Compared to arteries, the velocity of flow of the blood through the capillaries is A) impossible to predict without more information. B) much slower. C) at least twice as fast. D) about the same. E) at least 10 times faster.

B

7-24The cell that is the progenitor of all the types of blood cells is called the A) megakaryocyte. B) pluripotent hematopoietic stem cell. C) reticulocyte. D) committed progenitor cell. E) progenitor.

B

7-25Stretch-sensitive mechanoreceptors known as ________ are located in some artery walls. A) elasticeptors B) baroreceptors C) nociceptors D) chemoreceptors

B

7-27Capillaries are best described as A) thick walled vessels that carry blood rich in oxygen. B) microscopic vessels in which blood exchanges material with the interstitial fluid. C) thick walled vessels that convey blood away from the heart. D) thin walled vessels that convey blood toward the heart. E) thin walled vessels that carry blood deficient in oxygen.

B

7-8The process of fibrinolysis A) forms emboli. B) dissolves clots. C) draws torn edges of damaged tissue closer together. D) activates fibrinogen. E) forms thrombi.

B

8-11High carbon dioxide concentration in body fluids is called A) carbonation. B) hypercapnia. C) hypercarbia. D) hyperdioxia. E) hyperoxia.

B

9-19When venous return is increased, stretch receptors in the atria of the heart are activated. This results in A) increased glomerular filtration. B) inhibition of ADH secretion. C) secretion of ADH. D) increased thirst. E) decreased urine production.

B

9-28Which structure is NOT part of the blood circulation through the kidney? A) vasa recta B) loop of Henle C) glomerulus D) renal corpuscle

B

9-7The most important factor affecting the pH of plasma is the concentration of A) organic acids. B) carbon dioxide. C) ketone bodies. D) lactic acid. E) hydrochloric acid.

B

3Q48Summarize the steps of hemostasis.

1. Vasoconstriction--immediate constriction of damaged vessels caused by vasoconstrictive paracrines released by the endothelium 2. Temporary blockage of a break by a platelet plug--it mechanically blocks the hole with the platelet plug 3. Coagulation--the formation of a clot that seals the hole until tissues are repaired.

8-20 Match the type of breathing with its description. A. apnea B. hyperventilation C. tachypnea D. hyperpnea E. dyspnea 1. cessation of breathing 2. rapid breathing 3. difficulty breathing 4. increased respiratory rate and/or volume due to increased metabolism 5. increased respiratory rate and/or volume without increased metabolism

1=A apnea:cessation of breathing 2=C tachypnea:rapid breathing 3=E dyspnea:difficulty breathing 4=D hyperpnea:increased respiratory rate and/or volume due to increased metabolism 5=B hyperventilation:increased respiratory rate and/or volume without increased metabolism

1-10A study in which a participant acts as an experimental subject in part of the experiment and a control in another part of the experiment is called what? A) crossover study B) More than one of the answers is correct C) double-blind study D) meta-analysis study E) retrospective study

A

1-11Functions of connective tissue include A) All of the answers are correct. B) transporting fluids and dissolved materials. C) providing protection for delicate organs. D) establishing a structural framework for the body. E) storing energy reserves.

A

1-19Movement of material between cells is known as the ________ pathway. A) paracellular B) metacellular C) transcellular D) cisendothelial E) transendothelial

A

1-20The chemical bonding behavior of an atom is directly determined by A) the number of electrons. B) the number of neutrons. C) the number of protons. D) the mass of the atom. E) the size of the atom.

A

1-23All of the following are synthesized along various sites of the endoplasmic reticulum except one. Identify the exception. A) RNA B) proteins C) steroids D) fatty acids E) phospholipids

A

1-24Exocrine glands A) All of the statements are true. B) may make either mucous or serous secretions. C) may work as single cells or as a multicellular organ. D) release their secretions through open tubes, called ducts. E) release their secretions into the external environment.

A

1-8Which group of elements makes up more than 90% of the body's mass? A) O, C, H B) Ca, C, O C) C, Na, K D) O, Ca, H E) O, H, Na

A

3Q4Surfactant A) helps prevent the alveoli from collapsing. B) replaces mucus in the alveoli. C) is not found in healthy lung tissue. D) protects the surface of the lungs. E) phagocytizes small particulate matter.

A

4-19The brain area acknowledged as the center for emotions is the A) amygdala. B) pons. C) hypothalamus. D) cerebellum. E) suprachiasmatic nucleus.

A

7-11The lymphatic system A) empties the lymph vessels into the veins near the clavicles. B) All of the answers are correct. C) has heart-like pumps called lymph nodes. D) stores blood when circulatory demand is low. E) can be removed without health consequences.

A

7-15When blood pressure is normal, the receptors in arterial walls fire action potentials A) continuously. B) very seldom.

A

7-1Each of the following changes will result in increased blood flow to a tissue except one. Identify the exception. A) decreased vessel diameter B) increased blood volume C) increased blood pressure D) decreased peripheral resistance E) relaxation of precapillary sphincters

A

7-22Which of the following events result in the first heart sound? A) The AV valves close. B) The AV valves open. C) The atria contract. D) The semilunar valves close. E) The semilunar valves open.

A

7-7When the baroreceptor reflex is triggered by a decline in blood pressure, A) sympathetic activity increases and cardiac output increases. B) cardiac output increases. C) sympathetic activity increases. D) peripheral resistance decreases. E) sympathetic activity increases, cardiac output increases, and peripheral resistance decreases.

A

8-17An increase in the level of carbon dioxide in the blood will A) increase the rate of breathing. B) decrease the rate of breathing. C) decrease the alveolar ventilation rate. D) decrease pulmonary ventilation. E) increase the pH of arterial blood.

A

8-3Most of the carbon dioxide in the blood is transported as A) bicarbonate ions. B) solute dissolved in the cytoplasm of red blood cells. C) carbonic acid. D) carbaminohemoglobin. E) solute dissolved in the plasma.

A

8-6Ventilation is also known as A) breathing. B) air conduction. C) expiration. D) inspiration. E) blowing.

A

8-9Chronic hypoxia A) increases 2,3-DPG production in blood and can result from anemia. B) increases 2,3-DPG production in blood. C) can be caused by anemia. D) All of the answers are correct. E) shifts the HbO2 dissociation curve to the left.

A

9-11What effect would a decrease in pH have on the amount of potassium ion in the urine? A) a decrease in the amount of potassium in the urine B) an increase in the amount of potassium in the urine

A

9-12Water reabsorption by the kidneys is a result of A) osmosis. B) cotransport with ions. C) both passive and active transport processes. D) exchange with ions.

A

9-13Contents in the peritubular capillaries are actively transported into proximal and distal convoluted tubules in a process known as: A) secretion B) reabsorption C) filtration D) excretion

A

9-14Urine is produced by the A) kidney. B) urinary bladder. C) gallbladder. D) ureter. E) urethra.

A

9-1Cell volume (and therefore cell function) in most cells is dependent upon careful regulation of A) osmolarity of extracellular fluid. B) volume of extracellular fluid. C) blood pressure. D) resting membrane potential. E) permeability of cell membranes.

A

9-23Measurements in a nephron reveal a glomerular hydraulic pressure of 69 mm Hg, and a fluid pressure in the Bowman's capsule of 15 mm Hg. Assuming that the plasma osmotic pressure is 30 mm Hg, and that essentially no plasma proteins are filtered by the glomerulus, what is the net glomerular filtration pressure in this case? A) 24 mm Hg B) 114 mm Hg C) -6 mm Hg D) 54 mm Hg E) 84 mm Hg

A

9-24Which of the following statements about autoregulation is true? A) Myogenic response is the intrinsic ability of vascular smooth muscle to respond to pressure changes. B) In tubuloglomerular feedback, stretch-sensitive ion channels open, resulting in depolarization of smooth muscle cells. C) Myogenic response is a paracrine signaling mechanism. D) In myogenic response, the macula densa cells send a paracrine message to the neighboring afferent arteriole.

A

9-25The hormone that regulates water excretion by the kidneys A) only increases water permeability in certain portions of the kidney tubules. B) only decreases water permeability in certain portions of the kidney tubules. C) decreases water permeability throughout the kidney tubules. D) increases water permeability throughout the kidney tubules.

A

9-5In a normal kidney, which of the following conditions would cause an increase in the glomerular filtration rate (GFR)? A) a decrease in the concentration of plasma proteins in the blood B) an increase in the capsular hydraulic pressure C) constriction of the afferent arteriole D) a decrease in the hydraulic pressure of the glomerulus E) a decrease in the net glomerular filtration pressure

A

10M10=Describe types of energy expenditure

Body temp homeostatis is controlled by the hypothalamus. Heat loss occurs from radiation, conduction, convection & evaporation. It's promoted by cutaneous vasodilation & sweating. It is generated by shivering thermogenesis & by nonshivering thermogenesis (produces brown fat). Diet induced thermogenesis is an increase in heat production after eating. It is regulated w/i normal range (96-99 degrees F). It is homeostatically regulated.

7M3=Explain how flow and resistance are regulated in the cardiovascular system

By a pressure gradient, measured in mm HG. Blood flows from higher pressure to lower pressure. Pressure of fluid in motion decreases over distance.Flow through the tube is directly proportional to the pressure gradient. Resistance opposes flow. Blood flow takes the path of least resistance. Flow is inversely proportional to resistance. Determined by radius of the tube, length of the tube & viscosity (thickness) of the fluid

10-15Most products of fat digestion are absorbed by A) capillaries. B) arterioles. C) lymphatic vessels. D) veins.

C

10-18Which of the following is the sequence of layers from the lumen to the outer wall of the digestive tract? A) serosa, submucosa, mucosa, muscularis externa B) mucosa, submucosa, serosa, muscularis externa C) mucosa, submucosa, muscularis externa, serosa D) submucosa, muscularis externa, serosa, mucosa E) submucosa, mucosa, serosa, muscularis externa

C

10-1Nutrient absorption occurs primarily in theA) B) C) D) E) A) large intestine. B) stomach. C) small intestine. D) stomach and small intestine E) liver.

C

10-21Which of the following is an accessory organ of digestion? A) spleen B) esophagus C) pancreas D) stomach E) colon

C

10-24Functions of the large intestine include A) temporary food storage. B) chemical digestion of chyme. C) resorption of water and compaction of feces. D) All of these answers are correct. E) absorption of the products of digestion.

C

10-5An enzyme that will digest proteins into peptides is A) lipase. B) maltase. C) trypsin. D) amylase. E) nuclease.

C

10-8Chemical digestion refers to A) the absorption of nutrients in the gut. B) the effects of chewing, churning, and segmentation. C) the enzymatic breakdown of food. D) the input of food into the digestive tract. E) the progressive dehydration of indigestible residue.

C

1Q19An ion widely important in intracellular signaling is A) sodium. B) potassium. C) calcium. D) chloride. E) cobalt.

C

2-6A competitive inhibitor binds to A) a region of the enzyme other than the active site. B) the product. C) the active site. D) the surrounding tissue. E) the substrate.

C

3-2When stimulated by a particular hormone, there is a marked increase in the activity of G proteins in the membrane. The hormone is probably A) a steroid. B) estrogen. C) a peptide. D) aldosterone. E) testosterone.

C

3-3The pituitary hormone that controls the release of glucocorticoids from the adrenal cortex is A) LH B) TSH C) ACTH D) FSH E) STH

C

3Q10The QRS complex of an ECG corresponds to A) atrial repolarization. B) the depolarization of the atria. C) the progressive wave of ventricular depolarization. D) the repolarization of the ventricles. E) None of the answers are correct.

C

3Q13All of the following would cause an increase in blood pressure EXCEPT A) an increase in arterial resistance. B) sympathetic stimulation. C) a decrease in cardiac output. D) a decrease in arterial diameter.

C

3Q17The contraction cycle is triggered by the rise in ________ released from the sarcoplasmic reticulum (SR). A) K+ B) Na+ C) Ca2+

C

3Q24Flow of air A) is directly proportional to the resistance, and flow decreases as the pressure of the system increases. B) None of the answers are correct C) is directly proportional to a pressure gradient, and flow decreases as the resistance of the system increases. D) is directly proportional to the resistance, and flow increases as the resistance of the system increases. E) is directly proportional to a pressure gradient, and flow increases as the resistance of the system increases.

C

4-14Neurotransmitter is stored and released from A) axon terminals only. B) axon varicosities only. C) axon terminals and axon varicosities D) cell bodies only. E) dendritic spines only.

C

4-17What stops the rising phase of the action potential? A) The sodium activation gate closes. B) The K+ gate closes. C) The Na+ inactivation gate closes. D) The Na+ activation gate opens. E) The Na+ inactivation gate opens.

C

4-20Branches that sometimes occur along the length of an axon are called A) axon terminals. B) axon hillocks. C) collaterals. D) synapses. E) dendrites.

C

5-10Most preganglionic neurons originate in the thoracic and lumbar regions of the spinal cord. A) true for both divisions B) true only for the parasympathetic division C) true only for the sympathetic division

C

5-24A viral infection involving the vestibular nuclei may result in A) loss of sight. B) local paralysis. C) a sense of dizziness. D) high blood pressure. E) loss of hearing.

C

5-27The highest frequency sound is detected by A) hair cells located near the helicotrema end of the basilar membrane. B) the frequency at which the stereocilia vibrate. C) hair cells located near the oval window end of the basilar membrane. D) unique hair cells located anywhere along the basilar membrane. E) hair cells located near the middle of the basilar membrane.

C

5-30The adrenal medulla is closely allied with this system. A) true only for the parasympathetic division B) true for both divisions C) true only for the sympathetic division

C

5-31The neurotransmitter acetylcholine is released by neurons at the neuroeffector synapse. A) true only for the sympathetic division B) true for both divisions C) true only for the parasympathetic division

C

6-12The function of the titin protein is to A) cover the myosin binding site on the actin molecule. B) release calcium during contraction. C) stabilize the position of the contractile filaments. D) interact with actin during contraction. E) pull Z lines together during contraction.

C

6-14Compared to skeletal muscle, contraction of smooth muscle cells is A) a slower response to a stimulus. B) controlled by the somatic nervous system. C) a slower response to a stimulus and sustained without fatigue. D) sustained without fatigue. E) a slower response to a stimulus, sustained without fatigue, and controlled by the somatic nervous system.

C

6-2Most of the time, the parallel thick and thin filaments of the myofibrils are connected by ________ that span the space between myosin and actin molecules. A) nebulin molecules B) tropomyosin molecules C) crossbridges D) sarcomeres E) calcium ions

C

7-12The majority of the protein inside a red blood cell is A) immunoglobulin. B) porphyrin. C) hemoglobin. D) fibrinogen. E) albumin.

C

7-13 33. ________ are a group of diseases characterized by the abnormal growth and development of white blood cells, and ________ are diseases where patients have too few white blood cells. A) Neutropenias, anemias B) Anemias, leukemias C) Leukemias, neutropenias D) Neutropenias, leukemias E) Leukemias, anemias

C

7-17Plasma is mostly A) ions. B) organic molecules. C) water. D) proteins. E) blood cells.

C

7-19As blood vessel length increases, A) both resistance and flow increase B) resistance increases only. C) resistance increases and flow decreases. D) flow decreases only. E) friction decreases only.

C

7-23The only blood vessels whose walls permit exchange between the blood and the surrounding interstitial fluids are the A) venules. B) capillaries. C) venules and capillaries. D) arterioles and capillaries. E) arterioles.

C

7-2The P wave of an ECG corresponds to A) None of the answers are correct B) atrial repolarization. C) the depolarization of the atria. D) the repolarization of the ventricles. E) the progressive wave of ventricular depolarization.

C

4-21Clusters of nerve cell bodies in the peripheral nervous system are called A) nodes. B) microglia. C) glia. D) ganglia. E) neuroglia.

D

4-25The structure that connects the two cerebral hemispheres is the A) suprachiasmatic nucleus. B) gray "H." C) basal nuclei. D) corpus callosum. E) hippocampus.

D

5-11Which of the following would be an adequate stimulus for a mechanoreceptor? A) cold temperature B) oxygen C) photon of light D) cell stretch E) pH

D

5-13Put these structures of the visual system in the order that they transmit visual information from the retina. 1. optic disk 2. visual cortex of the occipital lobe 3. optic chiasm 4. lateral geniculate body of the thalamus A) 1, 2, 3, 4 B) 3, 1, 4, 2 C) 3, 1, 2, 4 D) 1, 3, 4, 2 E) 1, 3, 2, 4

D

7-14Restoring lost fluid from the capillaries back to the circulatory system is one of the major functions of the ________ system. A) digestive B) urinary C) thirst-quenching D) lymphatic E) immune

D

1-15The alpha-helix and pleated sheet are examples of the ________ structure of a protein. A) tertiary B) primary C) quaternary D) pentanary E) secondary

E

1Q10Atoms in a covalent molecule A) share electrons in single pairs. B) share electrons in double pairs. C) share electrons in triple pairs. D) share electrons singly, never in pairs. E) can share electrons in single pairs, double pairs, or triple pairs.

E

1Q15When an enzyme's activity is destroyed by heat or a change in pH, the enzyme is said to be A) broken. B) toxic. C) conjugated. D) tertiary. E) denatured.

E

1Q7An enzyme that transfers phosphates from ATP to a substrate molecule is a A) hydrolase. B) lipase. C) transferase. D) ligase. E) kinase.

E

2-27Information stored in the nucleus is translated into A) phospholipids. B) carbohydrates. C) lipids. D) None of the answers are correct. E) proteins

E

2-8Which of the following molecules can move across the phospholipid bilayer by simple diffusion? A) steroids B) water C) lipids and water D) lipids E) All of the answers are correct.

E

2Q11The region where the axon terminal meets its target cell is called the A) dendrites. B) nerve. C) hillock. D) collateral. E) synapse.

E

7-36The accumulation of fluid in the interstitial space is called ________.

EDEMA

8-26The ability of a lung to recoil, or recover from stretch, is called ________.

ELASTANCE

1-16Ribosomal RNA is formed by A) mitochondria. B) Golgi complexes. C) the endoplasmic reticulum. D) nucleoli. E) lysosomes.

D

2-3The addition of a phosphate group to a substrate is called ________. The enzyme that catalyzes this reaction is referred to as a ________. A) phosphorylation; phosphatase B) proteolysis; phosphatase C) proteolysis; kinase D) phosphorylation; kinase

D

2Q12Which statements apply to the parasympathetic division of the nervous system? A) It is dominant during "resting and digesting." B) All of the statements apply. C) Epinephrine is the primary neurotransmitter of the parasympathetic division. D) It is dominant during "resting and digesting" and its ganglia are nearby, on or near their target organs. E) Its ganglia are nearby, on or near their target organs.

D

2Q8Language and verbal skills tend to be ________ side of the brain of right-handed people. A) concentrated on the right B) equally distributed between the left and right C) randomly located with respect to the left or right D) concentrated on the left

D

3-1 The majority of hormones in the body are A) neurohormones. B) steroid hormones. C) All of the hormones are present in equal amounts in the body. D) peptide hormones. E) amino acid-derived hormones.

D

3-6The posterior pituitary gland secretes A) ACTH B) FSH. C) TSH D) ADH (Vasopressin).

D

3Q11Electrical shock to the heart is usually used to treat A) myocardial infarction. B) heart murmur. C) heart block. D) ventricular fibrillation. E) atrial fibrillation.

D

3Q19The most accurate definition of artery is a vessel that A) contains smooth muscle in its wall. B) transports blood toward the heart. C) contains internal valves. D) transports blood away from the heart. E) carries highly oxygenated blood.

D

8M7=Describe abnormal breathing patterns

Hypernia= increased respiratory rate &/or volume in response to metabolism (exercise) Hyperventilation = increased respiratory rate &/or volume NOT in response to metabolism (emotional or blowing up a balloon) Hypoventilation = decreased alveolar ventilation (shallow breathing, asthma, restrictive lung disease) Tachypnea = Rapid breathing; usually increased respiratory rate w/decreased depth (panting) Dyspnea = Difficulty breathing, described as "air hunger" (various pathologies or hard exercise) Apnea = Cessation of breathing (voluntary breath holding; depression of CNS control centers

6M6=Describe adaptations of skeletal muscle allowing the rapid generation of large amounts of ATP

Muscle fibers store energy for contraction in phosphocreatine. Anaerobic metabolism of glucose is a rapid source of ATP but is not efficient. Aerobic metabolism is very efficient but requires an adequate supply of oxygen to the muscles. Amt of ATP in a muscle at any 1 time = 8 twitches.

2Q43The gaps between adjacent Schwann cells on an axon are called ________.

NODES OF RANVIER

3M7=Explain how negative feedback loops regulate hormone release

In complex endocrine reflexes, hormones of the pathway act as negative feedback signals. when the last hormone in a pathway feeds back to supress secretion of its trophic (control the secretion of other hormones) hormones, the relationship is called long-loop negative feedback. In short-loop negative feedback, pituitary hormones feed back to decrease hormone secretion by the hypothalamus. Cortisol secretion.This keeps hormones in a pathway that stays w/i the range needed for an appropriate response. Feedback patterns are important in the diagnosis of endocrine pathologies.

10-34The ________ is a "housekeeping function" that sweeps food remnants and bacteria out of the upper GI tract and into the large intestine.

MIGRATING MOTOR COMPLEX

6-33In a ____ reflex, a sensory neuron synapses directly on a motor neuron.

MONOSYNAPTIC

3Q32The thick filament of the myofibril is composed of ________ molecules whereas ________ is a protein that makes up the thin filaments.

MYOSIN, ACTIN

2M8=explain how receptors can initiate changes in cell behavior

Many disase have been linked to defects in various aspects of signal pathways such as missing/defective receptors.

7M2=Describe the function of the pulmonary and systemic circuits

PULMONARY= from the right atrium, blood flows into the right ventricle to the pulmonary arteries where it is oxygenated. This turns from blue to red and then it goes to the pulmonary VEINS then the left atrium through the left ventricle out to the body through arteries and in to capillaries. SYSTEMIC=After leaving the capillaries, blood flows into the venous side moving from small veins to larger & larger. the veins from the upper part of the body form the superior vena cava & the veins from the lower body form the inferior vena cava; then they empty into the right atrium.

9-31The plasma concentration at which all of the renal carriers for a given substance are saturated is the ________.

RENAL THRESHOLD

6M8=Be able to describe the components of skeletal and smooth muscle reflexes

SKELETAL=controlled by CNS bcz somatic motor neurons always cause contraction in skeletal muscle. Reflex movements are integrated primarily in the spinal cord. Postural reflexes are integrated in the brain stem.

8-25A ________ is an instrument that measures the volume of air moved with each breath.

SPIROMETER

2M3=describe how DNA is "read" by cells to make proteins

Protein synthesis is controlled by nuclear genes made of DNA into a complementary base code on RNA. Alternative splicing of mRNA in the nucleus allows 1 gene to code for multiple proteins. mRNA leaves the nucleus and goes to the cytosol where, with the assistance of tRNA and rRNA, it assembles amino acids into a designated sequence. This process is called translation. Post translational modification converts the newly synthesized protein to its finished form.

1Q31mRNA is created in a process called __________________. It then leaves the ______ and enters the cytosol to direct ____________, the assembly of amino acids into protein.

TRANSCRIPTION, NUCLEUS, TRANSLATION

4M11=Describe the neurological basis of learning and memory

The limbic system is the center of emotion in the human brain.

4-3 1. Compare and contrast action and graded potentials. Your answer should include a definition of each, types, characteristics, ionic basis, functions, and anything else necessary to answer the question.

This is a lot of information that can be summarized as follows: "Graded potentials are short distance signals where action potentials are long distance signals" (Yahoo, 2008). Graded potentials: 1) Input signal 2) Usually occurs in dendrites and cell body 3) Involves mechanically, chemically or voltage-gated ion channels 4) Involves Na+, Cl-, Ca2+ ions 5) Depolarizing (Na+) or hyperpolarizing (Cl-) signals 6) Signal strength depends on initial stimulus; can be summed 7) Entry of ions through gated channels initiates the signal 8) Unique characteristics: a) no minimum level required to initiate; b) 2 signals coming close together in time will sum; c) initial stimulus strength is indicated by frequency of a series of action potentials Action potentials: 1) Regenerating conduction signal 2) Occurs in the trigger zone through axon 3) Involves voltage-gated channel ion channels 4) Involves Na+ and K+ ions 5) Depolarizing signal 6) Signal strength is all or none phenomenon; cannot be summed 7) Above-threshold graded potential at the trigger zone opens ion channels (signal initiation) 8) Unique characteristics: a) threshold stimulus required to initiate; b) refractory period: 2 signals too close together in time cannot sum. Graded potentials are variable-strength signals that travel over short distances and lose strength as they travel through the cell. They are used for short-distance communication. If a depolarizing graded potential is strong enough when it reaches an integrating region within a neuron, the graded potential initiates an action potential. Action potentials are very brief, large depolarizations that travel for long distances through a neuron w/o losing strength. This function is rapid signaling over long distances, such as from your toe to your brain (Silverthorn, p. 253). Graded potentials are local and occur in the dendrites and cell body or sometimes can occur near the axon terminals. In neurons of the CNS and efferent division, graded potentials occur when chemical signals from other neurons open chemically gated ion channels allowing ions to enter/leave the neuron. When a stimulus opens monovalent cation channels on the cell body of a neuron, Na ions move into the neuron bringing in electricity. The positive charge spreads as a wave of depolarization (local current flow—net positive electrical charge) through the cytoplasm like a stone thrown into water creates ripples that spread out from the point of entry. The strength of the initial depolarization in a graded potential is determined by how much charge enters the cell, just like the ripples depend on the size of the stone thrown into the pond. They will lose strength because of current leak (+ charges leak out into the cytoplasm) and cytoplasmic resistance (cytoplasm resists the flow of electricity). Depolarized graded potentials are considered excitatory and hyperpolarizing graded potentials are inhibitory. The ability of a neuron to respond to a stimulus and fire an action potential is called the cell's excitability. Examples of graded potentials are pacemakers, they are responsible for cardiac automaticity, postsynaptic membrane potentials and EPP end plate potential. Action potentials (also known as spikes) travel long distances. They are electrical signals of uniform strength that travel from a neuron's trigger zone to the end of its axon. Ion channels in the axon membrane open sequentially as electrical current passes down the axon. Additional Na+ entering the cell reinforce the depolarization, which is why it doesn't lose strength over distance like the graded potential. The action potential that starts it is exactly the same as the one in the trigger zone—a depolarization of about 100mV amplitude. They are all or none responses—either it occurs or it doesn't. The strength of the graded potential that initiates an action potential has no influence on the amplitude of the action potential. This is like the domino effect. The initiation of one triggers the next and the next, etc. The action potential is a wave of electrical energy that moves down the axon. Action potentials include nerve traffic, muscle contraction, hormone release, GI secretions and cognitive thought. They are required for sight, hearing and touch as they are dependent on action potentials for transmission of information to the brain. Graded potentials cause the generation of an action potential. They have three stages: 1) resting stage; 2) depolarization stage (NA+ ions flow into the cell as the threshold for voltage gated Na+ channels are exceeded); 3) repolarization stage (K+ ions flow out of the cell as voltage gated K+ channels are opened and the cell membrane potential moves back toward the resting membrane potential). The components of action potential: 1) threshold (when voltage gated channels open); 2) rising phase (as Na+ channels open, membrane potential begins to shift toward the equilibrium potential for Na+); 3) overshoot (when membrane potential becomes +); 4) peak (Na conductance begins to fall—the closure of the slow gate); 5) repolarization (inactivation of Na channels and opening of the K+ channels causes repolarization; 6) threshold (as membrane potential passes back through the threshold, the voltage gated channels reset (Na+ and K+ channels)). 7) After-hyperpolarization. Once an action potential has begun, a second cannot be triggered for about 1-2msec, no matter how large the stimulus (refractory period). The delay that allows for the Na+ channel gates to reset to their resting positions is called the absolute refractory period. Action potentials moving from trigger zone to axon terminal cannot overlap and cannot travel backward.

1-32Ions with a positive charge are called

cation // opposite = anion

2-34Name the three types of gated channels.

chemically gated, voltage gated, and mechanically gated

1-29A combination of two or more atoms that has physical and chemical properties that differ from the atoms that compose it is called a(n)

compound

1-30The molecule DNA contains the five-carbon sugar

dexoyribose

2M9=utilize independent research skills to answer questions about human disease

ok

3M2=Describe the chemical behavior of steroid hormones

synthesized as needed. Hydrophobic & bound to protein carriers. have an extended 1/2 life. steroid receptors are inside the cell.

1Q29Amino acids are carried to the ribosomes to be incorporated into polypeptide chains by

tRNA

6-22Asynchronous activation of motor units is important because it functions to

prevent fatigue

1-33Molecules that do not dissolve in water are called

hydrophobic // opposite = hydrophillic

111test

this is a test

2Q39The hormones of the anterior pituitary are controlled by trophic hormones from the ________.

HYPOTHALAMUS

3Q50Describe the process of relaxation of a skeletal muscle fiber.

Relaxation is an active process, resulting from loss of acetylcholine from the receptors as ACh is broken down and there is no further release. This allows the muscle cell to repolarize, which stops the release of calcium from the sarcoplasmic reticulum (SR). The pumping of calcium back into the SR causes the troponin to move the tropomyosin into a position that it allows only a low force crossbridge to form and the cell to relax.

2-32Carrier proteins operating at their maximum rate are said to be ___________.

SATURATED

7-33The ________ is a wall that separates the two sides of the heart.

SEPTUM

1Q30RNA is usually ________-stranded.

SINGLE

1-31The space that is surrounded by the tissue wall of hollow organs is known as the

lumen

8-23Match the factor with its effect on the affinity of hemoglobin for oxygen. A. increase B. decrease 1. increased temperature 2. increased pH 3. increased 2,3-DPG 4. increased PCO2

1=B 2=A 3=B 4=B

2Q37The outer layer of the adrenal gland is called the ________.

ADRENAL CORTEX

3-18The inner portion of the adrenal gland is called the

ADRENAL MEDULLA

10-36The enzyme that digests starch into disaccharides and trisaccharides is ________.

AMYLASE

2M4=understand why some molecules are able to cross the plasma membrane and others are not

Although the contents of the intracellular & extracellular compartments differ, homeostasis keeps them in a dynamic steady state. Movement of materials btwn & w/i compartments is necessary for como & is accomplished by bulk flow & biological transport. Flow of solutes and water across cell membranes occurs in response to osmotic, chemical (concentration) or electrical gradients. The selectively permeable cell membrane creates resistance to flow that can be overcome by changing the composition of the membrane lipids or by inserting membrane proteins that act as channels or transporters. Biological txport in the body requires energy from different sources: molecular motion, concentration gradients or chemical bonds. The binding of substrates to transporters demonstrates the theme of protein counteractions.

3Q38he superior portion of the heart where the major blood vessels enter and exit is the ________.

BASE

3Q40In smooth muscle cells, Ca2+ binds to ________ which activates the enzyme ________ to phosphorylate myosin and increase force.

CALMODULIN, MYOSIN LIGHT CHAIN KINASE (MLCK)

8-27The ease with which the lungs stretch in response to changes in pressure is termed ________.

COMPLIANCE

3Q35The dissolution of fibrin by plasmin is known as ________.

FIBRINOLYSIS

8-28An increase in the rate and depth of breathing is known as ________.

HYPERVENTILATION

10M6=Describe how the human body stores energy

It is stored in fat and glycogen. Our cells convert glucose to glycogen for storage & put it in the liver & skeletal muscles. Brain requires 150 g glucose/day. Body keeps most of its energy reserves in compact, high-energy fat molecules. Adipose tissue stores large amts of energy in minimal space. Metabolically, the energy in fat is harder to access & the metabolism of fats is slowre than carbs. Storage makes energy available for times of fasting.

3Q42Generally, PCO2 in arterial blood is ________ than PCO2 in venous blood

LESS THAN

10M5=Explain how patterns of smooth muscle contraction contribute to digestive function

Peristaltic contractions are progressive waves of contraction that occur mainly in the esophagus. Segmental contractions are primarily mixing contractions. GI motility is mediated by the enteric nervous system & modulated by hormones, paracrine signals & nneuropeptides. GI smooth muscle cells are electrically connected by gap junctions. Some segments are tonically contracted, but others exhibit phasic contractions. Intestinal muscle exhibits spontaneous slow wave potentials. When a slow wave reaches threshold, it fires action potentials & contracts. Slow waves originate in the interstitial cells. BOTTOM LINE = MOTILITY.

5M5=Explain how the ear transmits sound wave energy and creates the perception of sound

Sound transduction turns air waves into mechanical vibrations, then fluid waves, chemical signals & finally action potentials. The initial processing for pitch, loudness & duration of sound takes place in the cochlea. Localization of sound is a higher function that requires sensory input from both ears & sophisticated computation by the brain. The auditory pathway goes from cochlear nerve to medulla, pons, midbrain & thalamus before terminating in the auditory cortex. Info from both ears goes to both sides of the brain.

5M2=Describe the transmission and processing of sensory signals

Sensory transduction converts stimuli into graded potentials; A sensory neuron has a receptive field; The CNS integrates sensory info; coding & processing distinguish stimulus properties

3M1=List the chemical classes of hormones and be able to identify hormones as peptides, amines, or steroids

There are 3 types: 1) PEPTIDE/PROTEIN HORMONES=3 or more amino acids; 2) STEROID HORMONES=from cholesterol; 3) AMINO ACID DERIVED HORMONES=from tyrosine (thyroid) or trytophan (melatonin) MOST HORMONES ARE PEPTIDES OR PROTEINS

8M3=Explain the importance of alveolar surfactant

Type I alveolar cells = gas exchange. Type II alveolar cells = produce surfactant. Surfectant mixes w/the thin fluid lining of the alveoli to aid lungs as they expand during breathing. it decreases the work of breathing. creates surface tension, makes it easier to inflate.

9M7=Explain how the kidney contributes to pH homeostasis

pH affects intracellular proteins such as enzymes & membrane channels. Acid intake from foods & production by metabolic processes=body's biggest challenge to pH. Most significant source of acid is CO2 from respiration (combines w/water to form carbonic acid). Copes by changing pH w/buffers, ventilation & renal secretion or reabsorption of H+ and HCO3-. Bicarbonate produced from CO2 buffers organic acids produced by metabolism=most important extracellular buffer in body. Ventilation corrects bcz changes in plasma PCO2 dilutes when O is added. Acidosis= secrete H+ & reabsorb HCO3-. Alkalosis= secrete HCO3- & reabsorb H+

2-33At rest, nerve cells have a voltage of approximately _____ mV.

-70

1M10=Utilize independent research skills to answer questions about human disease

ok

4M5=Understand the function of glial cells

provide physical support and communicate with neurons. consists of Schwann cells (wraps around the axon & leaves little gaps called Nodes Of Ranvier between the myelin insulated areas) and satellite cells (non-myelating Schwan cells in the ganglia) in the peripheral nervous system and oligodendrocytes, microglia, astrocytes & ependymal cells in the CNS.

10-28Classify each scenario below as to primarily which type of work is being done. A. chemical work B. mechanical work C. transport work 1. a greyhound dog running around the track 2. wound healing, recovering from surgery 3. working on a dock, loading and unloading boxes into trucks all day 4. bringing glucose molecules inside brain cells so one can think 5. maintaining a concentration gradient across a membrane 6. enlarging one's muscles through body-building exercises

transport work:maintaining a concentration gradient across a membrane, mechanical work:working on a dock, loading and unloading boxes into trucks all day, chemical work:wound healing, recovering from surgery, chemical work:enlarging one's muscles through body-building exercises, transport work:bringing glucose molecules inside brain cells so one can think, mechanical work:a greyhound dog running around the track

6-21Calcium, released by the sarcoplasmic reticulum, binds to ________ to move the ________, which allows actin to form of a ________ with myosin.

troponin C, tropomyosin, high-force crossbridge

5M1=Describe different types of sensory receptors

1) CHEMORECEPTORS-chemical ligands (taste & smell) 2) MECHANORECEPTORS-various forms of mechanical energy (pressure, vibration, gravity, acceleration & sound) 3) THERMORECEPTORS-temperature 4) PHOTORECEPTORS-for vision, respond to light

2M5=distinguish between different types of membrane transport

1) DIFFUSION=bulk flow; a pressure gradient moves fluid & its dissolved/suspended materials. Movement depends on permeability of the membrane to that substance. Passibe txport uses no energy. Diffusion is passive down a chemical gradient that goes from higher to lower concentration. Stops at equilibrium. Simple diffusion inversely proportionate to membrane thickness. 2) PROTEIN-MEDIATED TXPORT=most mole's cross w/aid of membrane proteins. Channel proteins form water-filled channels. Gated channels regulate by open/closing. Carrier never form continuous connection btwn intra/extra cellular fluid. They bind to substrates then change conformation. Facilitated diffusion is protein-mediated (same properties as simple diffusion). Active Txport moves mole's agains concentration gradient & requires an outside source of energy. In primary, energy comes directly from ATP. 2ndary (indirect) active txport=uses potential energy stored in concentration gradient & is indirectly driven by energy from ATP. MOST IMPORTANT PRIMARY ACTIVE TRANSPORTER = SODIUM POTASSIUM-ATPase, that pumps Na+ out and K+ in. 3) VESICULAR TXPORT= large macromolecules & particles brought into cells by phagocytosis or endocytosis. Material leaves cells by exocytosis. When vesicles that come into the cytoplasm by endocytosis are returned to the cell membrane, the process is called membrane recycling. Exocytosis requires ATP. 4)EPITHELIAL TXPORT=allows 1-way movement of moles' across the epithelium. Move by paracellular or transcellular route. Larger molecules cross epithelia by transcytosis which includes vesicular transport. IN MOST LIVING CELLS, K+ IS THE PRIMARY ION THAT DETERMINES THE RESTING MEMBRANE POTENTIAL. CHANGES IN MEMBRANE PERMEABILITY TO IONS SUCH AS K+, Na+, Ca2+ OR Cl- ALTER MEMBRANE POTENTIAL AND CREATE ELECTRICAL SIGNALS.

8M1=List functions of the respiratory system

1) Exchange of gases between the atmosphere and the blood. (brings in O2 for distro & eliminates CO2 thru metabolism) 2) Homeostatic regulation of body pH. Can alter body pH by selectively retaining/excreting CO2. 3) Protection from inhaled pathogens & irritating substances. (trap & destroy potentially harmful substances before they can enter the body) 4) Vocalization. Bulk flow of air follows same principles as cardio stm: 1) flow takes place from regions of higher pressure to regions of lower pressure; 2) Muscular pump creates pressure gradients. 3) Resistance to air flow is influenced primarily by the diameter of the tubes that the air is flowing from. CELLULAR RESPIRATION IS THE INTRACELLULAR REACTION OF OXYGEN W/ORGANIC MOLECULES TO PRODUCE CARBON DIOXIDE, WATER & ENERGY IN THE FORM OF ATP. EXTERNAL RESPIRATION (THIS CHAPTER) IS THE MOVEMENT OF GASES BTWN THE ENVIRO & THE BODY'S CELLS. DONE BY VENTILATION (BREATHING), INSPIRATION (INHALATION) & EXPIRATION (EXHALATION)

1-36 1. Explain why the prefix homeo- is used in the term homeostasis. Why do some physiologists prefer the term homeodynamics over homeostasis?

1. Homeostasis is the ability of the body to maintain a relatively constant internal environment; it literally breaks down to mean "similar" + "condition". Walter Cannon wrote an essay in which he created the word homeostasis to describe the regulation of the body's internal environment. He use homeo rather than homo because the internal environment is maintained within a range of values rather than an exact fixed rate. Many scientists argue that we should use the word homeodynamics instead to reflect the small changes constantly taking place in our internal environment. The important concept to remember here is that the body monitors its internal state and takes action to correct disruptions that threaten its normal function.

1-35 1. Describe the structure of the cytoskeleton, and list its functions.

1. The cytoskeleton is a flexible, changeable 3-dimenstional scaffolding of the cell, composed of microfilaments, intermediate filaments and microtubules that extend throughout the cytoplasm. They cytoskeleton is structural support. It is changeable in nature. It has five important functions: 1) cell shape = provides mechanical strength and determines the shape of the cell & support microvilli; 2) internal organization = stabilize the position of the organelles; 3) intracellular transport = helps transports materials into the cell & within the cytoplasm by serving as an internal railroad track for moving organelles; 4) assembly of cells into tissues = allow the transfer of information from one cell to another; 5) movement = helps cells move.

1-34 1. What is the induced-fit model? List the types of bonds involved and classify them as strong or weak.

1. The induced-fit model is the active site changes shape to fit either substrate or product molecules. It is often referred to as the lock and key model. The substrate and the enzyme change shape so that they may bond to create the wanted products. The types of bonds involved are: ionic bonds (formed when oppositely charged ions are attracted to each other) and they are strong; hydrogen bonds (form when hydrogen atoms in polar molecules are attracted to O, N or Fl atoms. Hydrogen bonding among water molecules is responsible for the surface tension of water) and they are weak; Van der Walls forces (form when atoms are attracted to each other) are weak bonds.

1Q23Match the type of cell junction with the best description. Answers may be used once, more than once, or not at all A. gap junction B. tight junction C. anchoring junction 1. a cytoplasmic bridge 2. contributes to the blood-brain barrier 3. can be cell-matrix junctions 4. their loss is characteristic of cancer

1=A gap junction:a cytoplasmic bridge 2=B tight junction:contributes to the blood-brain barrier 3=C anchoring junction:can be cell-matrix junctions 4=C anchoring junction:their loss is characteristic of cancer

6-18Match the structures with the accurate description A. myofibril B. muscle fiber C. fascicle D. t-tubules E. sarcolemma 1. highly organized bundles of contractile proteins within a skeletal muscle cell 2. a single muscle cell 3. inward extensions of the muscle cell membrane 4. a bundle of adjacent muscle cells 5. the muscle cell membrane

1=A myofibril:highly organized bundles of contractile proteins within a skeletal muscle cell 2=B muscle fiber:a single muscle cell 3=D t-tubules:inward extensions of the muscle cell membrane, 4=C fascicle:a bundle of adjacent muscle cells 5=E sarcolemma:the muscle cell membrane

8-21Match the lung volume with its description. A. tidal volume B. expiratory reserve volume C. residual volume D. inspiratory reserve volume 1. the amount of air taken in during a single normal inspiration 2. the additional air inhaled after a normal inspiration 3. the minimum amount of air always present in the respiratory system, after blowing out all you can 4. the extra amount actively (forcibly) exhaled after a normal exhalation

1=A tidal volume:the amount of air taken in during a single normal inspiration 2=D inspiratory reserve volume :the additional air inhaled after a normal inspiration 3=C residual volume :the minimum amount of air always present in the respiratory system, after blowing out all you can 4=B expiratory reserve volume :the extra amount actively (forcibly) exhaled after a normal exhalation

5-32Match the response with the type of chemical. A. sympathetic agonist B. parasympathetic agonist 1. salivation 2. decreased activity in digestive tract 3. pupil dialation 4. fat breakdown

1=B 2=B 3=A 4=B

8-22Match the lung disease to its description. A. fibrotic lung disease B. pulmonary edema C. asthma D. emphysema 1. increased diffusion distance 2. destruction of alveoli 3. fluid accumulation in interstitial spaces 4. thickened alveolar membrane and decreased lung compliance 5. decreased surface area for gas exchange 6. increased airway resistance

1=B 2=D 3=B 4=A 5=D 6=C

7-29Match the following terms to the correct answer: A. bicuspid valve B. pulmonary valve C. aortic valve D. tricuspid valve 1. the semilunar valve on the right side of the heart 2. an AV valve that has three flaps 3. has three cuplike leaflets and has the aorta on one side 4. also called the mitral valve

1=B pulmonary valve:the semilunar valve on the right side of the heart 2=D tricuspid valve:an AV valve that has three flaps 3=C aortic valve:has three cuplike leaflets and has the aorta on one side 4=A bicuspid valve:also called the mitral valve

1Q24In the reaction shown, identify the role of each of the participants. Answers may be used once, more than once, or not at all. CO2 + H2O ---> H2CO3 A. enzyme B. substrate(s) C. product(s) 1. CO2 and H2O 2. carbonic anhydrase 3. H2CO3

1=B substrate(s):CO2 and H2O 2=A enzyme:carbonic anhydrase 3=C product(s):H2CO3

1Q22Match the organelle with its function. Answers may be used once, more than once, or not at all A. endoplasmic reticulum B. Golgi apparatus C. lysosomes D. mitochondria E. peroxisomes 1. The "powerhouse" of the cell, ATP is produced here. 2. These degrade long chain fatty acids and toxic foreign molecules. 3. It is the digestive system of a cell, degrading and/or recycling bacterial or organic components. 4. It modifies proteins and packages them into secretory vesicles for export from the cell.

1=D mitochondria:The "powerhouse" of the cell, ATP is produced here 2=E peroxisomes:These degrade long chain fatty acids and toxic foreign molecules 3=C lysosomes:It is the digestive system of a cell, degrading and/or recycling bacterial or organic components 4= B Golgi apparatus :It modifies proteins and packages them into secretory vesicles for export from the cell

2M6=describe common methods of cell-cell communication

2 basic types of physiological signals: chemical & electrical. Chem signals are basis for most como w/i the body. 4 methods: 1) direct cytoplasmic transfer through gap junctions (protein channels that connect 2 adjacent cells--when open signals pass directly from 1 cell to next); 2) contact-dependent signaling (require direct contact btwn surface mole's of 2 cells); 3) local chemical como; 4) long-distance como.

7M5=Describe the events of the cardiac cycle

2 phases: diasole (dialate--time of relaxation) systole (contracts--time when it contracts) 1. Heart at rest: atrial & ventricular diastole. 2. Completion of ventricular filling: atrial systole 3. Early ventricular contraction & the 1st heart sound. Blood pushing against eh underside of the AV valves forces them closed so that blood cannot flow back into the atria. Vibrations following closure of the AV valves create the 1st heart sound, the "lub" of "lub dub" 4. The heart pumps: ventricular ejection. 5. Ventricular relaxation & the second heart sound. the vibrations created by semilunar valve closure are the second heart sound, the "dub" of "lub dub".

1-34 2. Compare/contrast the chemical bonds between adjacent monomers in DNA, and between two strands of DNA.

2. DNA is deoxyribonucleic acid and is a double helix. A double helix is a three-dimensional structure that forms when two DNA chains link through hydrogen bonds between complementary base pairs. Deoxyribose is the sugar in the backbone and the four bases are adenine, guanine, cytosine and thymine. The hydrogen bonding follows specific rules: 1) because purines are larger than pyrimidines, space limitations always pair a purine with a pyrimidine; 2) guanine forms three hydrogen bonds with cytosine; 3) adenine forms two hydrogen bonds with thymine or uracil. More energy is required to break the triple hydrogen bonds of guanine and cytosine than the double bonds of adenine and thymine or adenine and uracil.

1-35 2. Describe the composition and function of the extracellular matrix.

2. Extracellular matrix (usually just called Matrix) is material that is synthesized and secreted by the cells of a tissue. Scientists used to believe that its only function was to hold cells together. Evidence now shows that the matrix plays a vital role in many physiological processes, ranging from growth and development to cell death. Many disease states are associated with overproduction or disruption of matrix including heart failure and the spread of cancer cells in the body. The composition of matrix varies from tissue to tissue and with elasticity and flexibility. It has two basic components: proteoglycans (glycoproteins covalently bound to polysaccharide chains); and insoluble protein fibers (collagen, fibronectin and laminin that provide strength and anchor cells to the matrix). The amount of matrix in a tissue is highly variable. The consistency can vary from watery (blood) to rigid (bone).

1-36 2. Explain why animals are used in research. Are there any limitations to the application of animal data to human physiology? Could these limitations be addressed using cell or tissue culture, or computer simulations?

2. I found this answer online, and I'm having difficulty putting it in to my own words. The answer states, "How human organ systems perform may be different in very subtle ways from corresponding systems in other species. Cells in culture are in an artificial environment, and while much has been learned from such systems, it has also been noted that the behavior of cells in culture is not identical to cells in a living body. Furthermore, cells cultured from established lines can change over time, becoming less like the original cells from which they were derived, and presumably less like normal cells. Computer simulations are valuable, but are only as good as the data entered, and given that we don't know everything there is to know about physiology, we can't write a perfect computer program. All three approaches are useful, but for different reasons, and therefore one research system does not completely substitute for another, nor is it appropriate to abandon one entirely". "Ethical questions arise when humans are used as experimental subjects" and I would like to add that the same ethical questions should be asked when living animals are used as experimental subjects. For example, is botox that is injected to temporarily lessen the appearance of wrinkles really worth the hundreds of animals that have to die for each batch?

1-36 3. Provide an example of a control system. Be sure to include the three main components: an input signal, a controller, and an output signal.

3. Regulated variables are kept within their acceptable (normal) range by physiological control mechanisms that kick in if the variable ever strays too far from its set point, or optimum value. There are two basic patterns of control mechanisms: local control and long-distance reflex control. In their simplest form, all control systems have three components: 1) output signal; 2) controller or integrating center; 3)Output signal. There is a figure in the text that illustrates a simple control system as: Input signal --> Integrating Center --> Output Signal --> Response An example would be: Oxygen concentration in a tissue decreases. Cells lining the small blood vessels that bring blood to the area sense the lower oxygen concentration (input signal) and respond by secreting a chemical signal. The signal molecule diffuses to nearby muscles in the blood vessel wall (integrating Center) bringing them a message to relax (output signal). Relaxation of the muscles widens the blood vessel, which increases blood flow into the tissue and brings more oxygen to the area (Response).

1-35 3. How do fully differentiated cells differ from stem cells? How can stem cells be used in medical treatment? Give examples.

3. Stem cells are immature cells that have the ability to differentiate. As stem cells differentiate, they narrow their potential fates and become pluripotent. Pluripotent cells can develop into many different cell types but not all cell types. An isolated pluripotent cell cannot develop into an organism. As differentiation continues, pluripotent cells develop into the various tissues of the body. As the cells specialize and mature, many lose the ability to undergo mitosis and reproduce themselves. They can be replaced, however, by new cells created from stem cells, less specialized cells that retain the ability to divide. Undifferentiated stem cells are said to be multipotent. An example of multipotent stem cells are bone marrow and give rise to blood cells. Stem cells can be used in medical treatment of degenerative diseases such as Parkinson's, heart attack or stroke. Stem cells could be implanted to treat damaged tissues. Stem cells come from embryonic and fetal cells, therefore, there is much controversy surrounding their use.

1-34 3. Water striders are insects that literally walk on water. These insects are frequently found living on ponds. If hydrogen bonds did not exist, how would this affect the life of water striders?

3. f hydrogen bonds did not exist, then water striders would not be able to walk on water; they would sink. Hydrogen bonds form between a hydrogen atom and a nearby oxygen, nitrogen or fluorine atom. So, for example, the polar regions of adjacent water molecules allow them to form hydrogen bonds with one another. The water striders would have several options if the hydrogen bonds did not exist. For example, they could learn to swim; they could adapt to live under water; they could learn to fly or jump to catch their prey; or they would simply die. Regardless, they would have to adapt and evolve.

1-34 4. Ionic bonds are considered to be strong chemical bonds. Yet, ions dissociate in water. Explain this apparent contradiction.

4. Ionic bonds are considered to be strong chemical bonds. Yet, ions dissociate in water. In my own words and as I understand it I will give an attempt to explain this contradiction. Since the textbook has offered very little direct information on nearly every question this worksheet has asked, I can "best guess" that ionic bonds dissociate in water because even though these noncovalent bonds are strong and share an electron, they are still strong in water, but water is also a noncovalent ionic bond. Therefore, the water and the salt keep pulling one or more electrons from each other. The strong ionic bond still exists, but it is fluid between the salt and the water; it bounces back and forth—as I try to describe it visually.

1-36 4. Write a teleological explanation for why heart rate increases during exercise. Now write a mechanistic explanation for the same phenomenon

4. The teleological approach to science explains the "why" something works. The mechanistic approach tells the "how". Students often confuse the two approaches because instructors ask WHY a physiological event occurs when they really want to know HOW it occurs. Staying aware of the two approaches will help prevent confusion. Therefore, the teleological explanation for why heart rate increases during exercise is because your body needs more oxygen and your heart has to beat quicker to provide it. The mechanistic explanation for HOW your heart rate increases during exercise is as your intensity of exercise increases, your heart rate will increase to match it.

1-34 5. You are a student intern in the research and development department of a pharmaceutical company. You have discovered a compound that destroys the common cold virus in cultured human cells. Chemical characterization reveals that carbon, hydrogen, and oxygen are present, in a 20:40:4 ratio of C:H:O. What chemical class is this compound? Experiments in rats show that neither oral nor injectable treatment with the compound was effective in destroying the virus. Discuss some possible reasons for this lack of effectiveness.

5. If I were working for a pharmaceutical company, which I would not, and I discovered a cure for the common cold that consisted mostly of carbon and hydrogen then I would be a lipid. Lipids are biomolecules made mostly of carbon and hydrogen. An important characteristic of lipids is that they are nonpolar and therefore not very soluble in water. Lipids can be divided into two broad categories: fats (solid at room temperature & derived from animal sources); oils (liquid at room temperatures & most plant lipids are oils). Experiments in rats showed that neither the oral or injectable treatment with the compound was effective in destroying the virus possibly because: 1) blood is similar to water & the lipids do not transport well in water; 2) just because the treatment does not work in rats, does not mean that it will not work in humans since our biological make up is different; 3) a petri dish may offer a better growing or killing medium than an actual body.

1-3Chemical reactions that occur in the human body proceed at a faster rate due to special catalytic molecules called A) enzymes. B) None of the answers are correct. C) activators. D) cytozymes. E) cofactors.

A

1-4Functions of epithelia include all of the following EXCEPT A) storing energy reserves. B) controlling permeability. C) producing specialized secretions. D) movement of substances. E) providing physical protection.

A

10-10Metabolism is a term that describes A) all chemical reactions that take place within an organism. B) all work done by a living organism. C) the extraction of nutrients from biomolecules. D) the energy released from chemical bonds in living cells. E) only chemical reactions that release ATP from living cells.

A

10-13Heat loss is promoted by A) sweating and dilation of cutaneous blood vessels. B) sweating, dilation of cutaneous blood vessels, and nonshivering thermogenesis. C) sweating. D) nonshivering thermogenesis. E) dilation of cutaneous blood vessels.

A

10-26During the absorptive state, A) the liver forms glycogen. B) All of these answers are correct. C) adipocytes contribute fatty acids to the circulation. D) skeletal muscles break down glycogen. E) glucagon levels are elevated.

A

10-4Lipoproteins that carry mostly cholesterol and phospholipids from peripheral tissues to the liver are called A) high-density lipoproteins (HDLs). B) very low-density lipoproteins (VLDLs). C) low-density lipoproteins (LDLs). D) intermediate-density lipoproteins (IDLs). E) very high-density lipoproteins (VHDLs).

A

10-7Rugae, plicae, and villi are all features that accomplish A) an increase in surface area. B) mechanical digestion. C) secretion of enzymes. D) immune function. E) secretion of hormones.

A

1Q12Enzymes increase reaction rate by A) lowering the activation energy of a reaction. B) increasing the free energy in the products. C) raising the activation energy of a reaction. D) converting an endergonic reaction to an exergonic reaction. E) None of these choices are correct.

A

1Q17Pinocytosis is a type of A) endocytosis. B) exocytosis. C) phagocytosis. D) endocytosis and exocytosis. E) exocytosis and phagocytosis.

A

2-1Which of the following is NOT true of diffusion in the human body? A) Smaller molecules take longer to diffuse than larger ones. B) Diffusion occurs faster at higher temperatures. C) Diffusion is rapid over short distances and slower over longer distances. D) Net movement of molecules occurs until the osmolarity is equal.

A

2-22How would the lack of a required cofactor for an enzyme affect that enzyme's function? A) The enzyme would not be able to function. B) The enzyme's function would not be altered. C) The enzyme would function more slowly. D) The enzyme would function more quickly.

A

2-23Typically, when steroid hormones bind to their receptors, A) gene transcription will increase or decrease. B) adenylyl cyclase is activated. C) G proteins are inhibited. D) protein kinases are activated. E) cyclic nucleotides are formed.

A

2-7When you eat a large meal and your body absorbs a lot of glucose and that makes its way to the interstitial fluid before going into the cell. 100% of the glucose should be absorbed into the cell from the interstitial fluid. Why does nearly all of the glucose enter the cell, rather than only half of it? A) It is modified by the cell, so there is still more glucose on the outside of the cell than inside it. B) Insulin forces glucose into the cell against a concentration gradient. C) The cells make ATP so fast, they use up all the glucose as soon as it enters the cell. D) It is moved by active transport.

A

2Q10Phasic receptors A) are quick to adapt to a particular stimulus intensity. B) once threshold is reached by a stimulus will always generate action potentials C) slowly decrease the frequency of action potentials generated to a constant stimulus. D) are attune to parameters that the body must continuously assess.

A

2Q13Autonomic motor neurons are subdivided into the A) sympathetic and parasympathetic divisions. B) somatic and enteric divisions. C) central and peripheral divisions. D) visceral and somatic divisions. E) visceral and enteric divisions.

A

2Q14The central opening in the eye through which the light passes on to the retina is the A) pupil B) fovea C) cornea D) lens E) conjunctiva

A

2Q16The falling phase of the action potential is due primarily to A) K+ flow out of the cell only. B) Na+ flow out of the cell and K+ flow into the cell. C) Na+ flow out of the cell only. D) Na+ flow in the cell only. E) K+ flow into the cell only.

A

2Q19There are ________ primary taste (gustation) sensations. A) 5 B) 20 C) 400 D) 2 E) 4

A

2Q1Which brain area is considered to be a key integrating center for homeostasis? A) hypothalamus B) thalamus C) pituitary gland D) medulla E) brain stem

A

2Q21Which of the following is NOT a somatosensory modality? A) vision B) proprioception C) nociception D) touch E) temperature

A

2Q25Inhibitory postsynaptic potentials (IPSPs ) A) result in local hyperpolarizations. B) prevent the escape of potassium ions. C) increase membrane permeability to sodium ions. D) result in local depolarizations. E) prevent the escape of calcium ions.

A

2Q33Match the description with the drug type: Blocks secretion of adrenal catecholamines A) sympathetic antagonist B) parasympathetic antagonist

A

2Q36Match the description with the drug type: Blocks sweating A) sympathetic antagonist B) parasympathetic antagonist

A

2Q4Each of the following hormones is an amino acid derivative except one. Identify the exception. A) thyroid-stimulating hormone B) norepinephrine C) epinephrine D) melatonin E) thyroid hormone

A

3-7Synergism occurs when A) hormones working together produce a larger effect than predicted. B) one hormone triggers the secretion of a second hormone. C) a hormone can exert its full effects only in the presence of another hormone. D) hormones working together produce a smaller effect than predicted. E) one hormone inhibits the release of a second hormone.

A

3Q12When a skeletal muscle generates enough force during contraction to shorten, A) only the sarcomere will shorten. B) the sarcomere and actin will shorten. C) only the actin will shorten. D) the sarcomere, actin, and myosin will shorten. E) only the myosin will shorten.

A

3Q14The function of red blood cells is to A) carry oxygen from the lungs to the body's cells. B) remove carbon dioxide from the lungs. C) remove nitrogenous wastes from active tissues. D) carry nutrients from the digestive system to the body's cells. E) defend the body against infectious organisms.

A

3Q18Most reflex movements are integrated by A) the spinal cord. B) central pattern generators. C) the brain. D) effectors. E) proprioceptors.

A

3Q25Which fibers fatigue sooner? A) fast-twitch fibers B) slow-twitch fibers

A

3Q26The porphyrin ring of heme contains an atom of A) iron. B) magnesium. C) copper. D) calcium. E) sodium.

A

3Q28-14. The depolarization of the pacemaker action potential spreads to adjacent cells through A) gap junctions. B) desmosomes. C) chemical synapses. D) tight junctions.

A

3Q5If a person inhales as deeply as possible and then blows the air out until she cannot exhale any more, the amount of air that she expelled is her A) vital capacity. B) inspiratory reserve volume. C) minimal volume. D) expiratory reserve volume. E) tidal volume.

A

4-29. The total amount of neurotransmitter released at the axon terminal is directly related to A) the total number of action potentials. B) the amplitude of the graded potential. C) the length of the axon. D) the amplitude of the action potential. E) None of these answers are correct

A

4-32What does the Goldman-Hodgkin-Katz equation take into account that the Nernst equation does NOT? A) the permeabilities of the ions B) the sizes of the ions C) the solubilities of the ions D) the electrical charges of the ions E) the temperature

A

4-5Graded potentials may A) All of the statements are true. B) be called EPSPs or IPSPs. C) initiate an action potential. D) hyperpolarize the membrane. E) depolarize the membrane to the threshold voltage.

A

5-1 Convergence describes A) how multiple primary sensory neurons synapse on a single secondary neuron. B) None of these choices are correct C) how receptive fields can overlap with one another. D) the reason why receptive fields are so small. E) how one primary sensory receptor neurons synapses with a secondary neuron.

A

5-16Nociceptors are responsible for the perception of A) pain and itch. B) pain only. C) pain and skin stretch. D) itch only. E) skin stretch only.

A

5-17It releases norepinephrine at the neuroeffector synapse. A) true only for the sympathetic division B) true for both divisions C) true only for the parasympathetic division

A

5-26Tonic receptors A) slowly decrease the frequency of action potentials generated to a constant stimulus. B) are quick to adapt to a particular stimulus intensity. C) are attune to changes in a parameter, not the extent of that change. D) All of these answers are correct. E) will always decrease to the point where no action potentials are generated.

A

5-28On average, one preganglionic neuron synapses with eight or nine postganglionic neurons, each innervating a different target. A) true for both divisions B) true only for the parasympathetic division C) true only for the sympathetic division

A

5-2The cell bodies of preganglionic neurons are found either in the brain stem or in the sacral region of the spinal cord. A) true only for the parasympathetic division B) true for both divisions C) true only for the sympathetic division

A

5-3The ganglia are found in a chain that runs close to the spinal cord or along the descending aorta. A) true only for the sympathetic division B) true for both divisions C) true only for the parasympathetic division

A

6-11Which fibers generate more force? A) fast-twitch fibers B) slow-twitch fibers

A

6-13 During heavy exercise, the ATP requirements of active muscle are likely to be met by metabolism of A) carbohydrates B) protein C) fatty acids. D) nucleotides

A

6-17After death, when metabolism stops, in which step of the contractile cycle must skeletal muscles remain? A) the rigor state B) the power stoke phase C) It depends on what part of the contractile cycle they were in at the time of death. D) a weak binding state

A

6-1Which of the following is a characteristic of slow-twitch oxidative skeletal muscle fibers? A) long contraction duration and high capillary density B) long contraction duration, few mitochondria, and high capillary density C) long contraction duration only D) high capillary density only E) few mitochondria only

A

6-4________ is the backup energy molecule that can be rapidly converted to ATP in active skeletal muscle. A) Phosphocreatine B) Glucose C) Protein D) Fatty acid

A

7-26When the heart is in fibrillation, A) effective pumping of the ventricles ceases because the myocardial cells fail to work as a team, and the brain cannot get adequate oxygen. B) the myocardial cells may become damaged from contracting too fast. C) the myocardial cells deplete their oxygen supply because they are contracting too fast, and the lactic acid produced damages the myocardial cells. D) the myocardial cells are contracting together as they should; fibrillation indicates a normal sinus rhythm of 75 beats per minute. E) there is no contraction of the myocardium.

A

8-10When the diaphragm and external intercostal muscles contract, A) the volume of the thorax increases. B) the volume of the lungs decreases. C) expiration occurs. D) the lungs collapse. E) the volume of the thorax decreases.

A

8-14Joe is playing in an intramural football game when he is tackled so hard that he breaks a rib. He can actually feel a piece of the rib sticking through the skin, and he is having a difficult time breathing. Joe probably is suffering from A) a pneumothorax. B) an obstruction in the bronchi. C) a collapsed trachea. D) decreased surfactant production. E) a bruised diaphragm.

A

1Q25Carrier proteins, channels, and vesicular transport are all ways to import molecules into a cell. Compare and contrast these three forms of transport, and explain why a cell needs so many different ways to import molecules.

A cell needs so many different ways to import molecules because some molecules need to rapid absorption, some need slow absorption. Additionally, in order to maintain homeostasis, the cell needs to regulate the molecules that come into and out of the cell. Many different transport methods are needed in order to bring certain molecules in at certain rates, according to the needs of the cell. A lot of things are happening at the same time, diffusion and osmosis, and many different types of transport are needed in order to answer the needs of the cell and their concentration gradients. Carrier proteins bind to the substrates that they carry but never form a direct connection between the intracellular fluid and extracellular fluid. Channels create water-filled passageways that directly link the intracellular and extracellular compartments. Vesicular transport is the movement of vesicles within the cell with the aid of the cytoskeleton. Vesicular transport

3M6=Describe common types of hormone receptors

A hormone receptor is a molecule that can bind to a specific hormone. Receptors for peptide hormones tend to be found on the plasma membrane of cells, whereas receptors for lipid-soluble hormones are usually found within the cytoplasm. Upon hormone binding, the receptor can initiate multiple signaling pathways which ultimately lead to changes in the behaviour of the target cells. 1. Water-soluble hormones include glycoproteins, catecholamines and peptide hormones composed of polypeptides, e.g. thyroid-stimulating hormone, follicle-stimulating hormone, leutinizing hormone and insulin. These molecules are not lipid-soluble and therefore cannot diffuse through cell membranes. Consequently, receptors for peptide hormones are located on the plasma membrane. The main two types of transmembrane hormone receptor are the G-protein-coupled receptor and the enzyme-coupled receptor. These receptors generally function via intracellular second messengers, including cyclic AMP (cAMP), cyclic GMP (cGMP), inositol 1,4,5-trisphosphate (IP3) and the calcium (Ca2+)-calmodulin system. 2. Lipid-soluble hormone receptors Steroid hormone receptors and related receptors are generally soluble proteins that function through gene activation. Their response elements are DNA sequences (promoters) that are bound by the complex of the steroid bound to its receptor. The receptors themselves are zinc-finger proteins.[1] These receptors include those for glucocorticoids, estrogens, androgens, thyroid hormone (T3), calcitriol (the active form of vitamin D), and the retinoids (vitamin A).

7-39 2. Describe what a nurse hears through the stethoscope while using a blood pressure cuff, and explain the significance of the sounds.

A sphygmomanometer and a stethoscope are used to measure blood pressure. First, the cuff is inflated so that it stops arterial blood flow. No sound can be heard through the stethoscope placed over the brachial artery at this time. Second, As the pressure is slowly released through the sphygmomanometer, Korotkoff sounds can be heard by pulsatile blood flow through the compressed artery (she hears thuds). Third, when the cuff pressure has reached a certain pressure through the final stages of released compression of the brachial artery, blood flow is silent. Guidelines published in the 2003 JNC 7 report now recommend that individuals maintain their blood pressure below 120/80.

3-9Match each hormone with its primary source. A. epinephrine B. aldosterone C. insulin D. melatonin E. calcitonin F. prolactin 1. adrenal medulla 2. anterior pituitary 3. pineal 4. adrenal cortex 5. thyroid 6. pancreas

A=1 epinephrine:adrenal medulla, C=6 insulin:pancreas, E=5 calcitonin:thyroid, F=2 prolactin:anterior pituitary, B=4 aldosterone:adrenal cortex, D=3 melatonin:pineal

3-8Match each term with the appropriate description A. peptide hormone B. hypothalamus C. steroid hormone D. amino acid-derived hormone E. anterior pituitary F. posterior pituitary 1. site of vasopressin synthesis 2. derivatives of tyrosine 3. lipophobic molecule that interacts with receptors on cell surface 4. cannot be stored in secretory vesicles 5. true endocrine gland of epithelial origin 6. storage and release site for oxytocin

A=3 peptide hormone: lipophobic molecule that interacts with receptors on cell surface , B=1 hypothalamus: site of vasopressin synthesis , E=5 anterior pituitary: true endocrine gland of epithelial origin , C=4 steroid hormone: cannot be stored in secretory vesicles , D=2 amino acid-derived hormone: derivatives of tyrosine , F=6 posterior pituitary: storage and release site for oxytocin

4-35The time during which an excitable membrane cannot respond to further stimulation regardless of the stimulus strength is the

ABSOLUTE REFRACTORY PERIOD

3-14The adrenal cortex produces a steroid hormone called _______________ that controls Na+ and K+ homeostasis and and another steroid hormone called ______ that controls blood glucose levels.

ALDOSTERONE; CORTISOL

5M7=Explain what "antagonistic control" means

ANTAGONISTIC CONTROL IS A HALLMARK OF THE AUTONOMIC DIVISION. One branch is excitatory and the other branch is inhibitory. So, the sympathetic side increases heart rate while the parasympathetic side decreases heart rate.

2Q41Two hormones that have opposing effects are called ________.

ANTAGONISTS

1Q28During the synthesis of proteins, amino acids are assembled in the proper sequence because the tRNA molecules that bring them to the ribosome have a(n) _______________ that is complementary to a specific codon in the mRNA.

ANTICODON

7-31The inferior point of the heart is called the ________.

APEX

7-40 3. Compare and contrast the structure and location of arteries, arterioles, capillaries, and veins. How do these differences contribute to the differences in function of each?

ARTERIES = Blood vessels that carry blood away from the heart to the arterioles. ARTERIOLES = The smallest arteries and site of variable resistance in the circulatory system. they are downstream from the arteries that direct distribution of blood flow to individual tissues by selectively constricting and dilating (site of variable resistance). CAPILLARIES = The smallest blood vessels where blood exchanges material with the interstitial fluid. Their leaky epithelium allows exchange of materials between the plasma, the interstitial fluid and the cells of the body. VEINS = Blood vessels that return blood to the heart located at the distal end of the capillaries. they act as a volume reservoir from which blood can be sent to the arterial side of the circulation if blood pressure falls too low. Blood flows back to the heart from here. Arteries and arterioles carry blood away from the heart. Veins and capillaries carry blood back to the heart.

3Q45In order to contract and relax, muscle cells require energy in the form of ________. The immediate backup energy source is ________ that requires the enzyme ________.

ATP, PHOSPHOCREATINE, CREATINE KINASE

6-32The heavy chain of myosin contains 2 important regions: the first acts as ________ to convert energy into movement whereas the second binds to ________.

ATPase, ACTIN

6-20The heavy chain of myosin contains 2 important regions: the first acts as ________ to convert energy into movement whereas the second binds to ________.

ATPase, actin

3-17When an organ or gland shrinks due to lack of use, that condition is referred to as

ATROPHY

10-44 3. Calculate the body mass index of a woman who is 5'5" tall and weighs 180 lbs. Now calculate the BMI of a man with those dimensions. 1 kg = 2.2 lbs, 1 m = 39.24 in. Is either individual considered obese? Explain. Could BMI be misleading as a means of indicating obesity? Explain.

According to the textbook on page 740, BMI is calculated as: weight (lb) x 703/height2 (in) = BMI 180 x 703/4225 = 180 x .1663 = Total = 29.95 that rounds to 30 BMI. Both are considered obese. BMI can be misleading as a means of indicating obesity. Also according to the text the following paragraph is true: a BMI between 18.5 - 24.9 is considered normal weight. The BMI calculation does not distinguish between fat mass and muscle mass, however, and heavily muscled athletes, such as football players, may have a BMI that seems unhealthy. Muscle tissue weighs more than fat, which explains the discrepancy. BMI calculations also do not allow for differences due to age, gender and ethnicity. For example, data indicate that Asians with BMIs in the normal weight range may still be at risk of certain diseases. Researchers have suggested that a fat mass index (fat mass/height2) is a better health indicator than BMI. I am in the military and this is not how we calculate BMI. Additionally, we have specific values and ranges for determining what a safe BMI calculation is.

2-35 5. The body maintains pH of all body fluids within a narrow range. Why is pH homeostasis important and what would happen if it were not maintained?

Acid production is the major challenge to pH homeostasis in the body. There are three ways the body copes with pH balance: buffers in the cells and plasma, the lungs and the kidneys. Through each of these, the end goal is to create a pH balance between 7.38-7.42, on a scale of 1.0 (acid) to 14.0 (alkaline), with a neutral of 7.0. Proper pH keeps the body system clean and functioning. When the body can't neutralize and eliminate acids, it sticks it in the extracellular fluid and connective tissues. This compromises cellular activity and will eventually corrode body tissue creating degenerative disease. In the process of the acid breakdown of the tissues, opportunists like bacteria, fungi and yeast move in. On top of the acid waste that the body is unable to eliminate, the bacteria, fungi and yeast create their own waste products that results in more toxicity of the body. At this point, many people will turn to Western medicine and seek the aid of pharmaceuticals to help correct the imbalance. More often than not, the pharmaceuticals create acidic conditions within the body that then intensifies the toxicity rather than helping to eliminate the problem. If the pH is not returned to normal by doing things such as eliminating acidic foods and beverages, increasing water intake and exercise then the body will degrade into life threatening illness such as cancer. The end result of not maintaining pH homeostasis will eventually lead to death.

4M2=Describe how action potentials are generated and propagated

Action potentials are rapid electrical signals that travel undiminished in amplitude (strength) down the axon from the cell body to the axon terminals. They being in the trigger zone. They are uniform, all-or-non depolarizations that can travel undiminished over long distances. The rising phase of the action potential is due to increased Na+ permeability. The falling phase is due to increased K+ permeability. The voltage-gated Na+ channels of the axon have a fast activation gate and a slower inactivation gate. Very few ions cross the membrane during an action potential. The Na+ -K+ -ATPase eventually restores Na+ and K+ to their original compartments. Once it has begun, ther eis a brief period of time known as the absolute refractory period when a 2nd action potential cannot be triggered, no matter how large the stimulus. Bcz of this, action potentials cannot be summed. During the relative refractory period, a higher-than-normal graded potential is required to trigger an action potential. Larger-diameter axons conduct action potentials faster than smaller diameter axons do.

1Q26Stem cell research has become a political topic in the last few decades. Explain why the research is being done, why some cells are favored for this research over others, and what the factors are that cause some people not to support this type of research. Are any alternatives available that are less opposed?

All cells in the body are derived from the single cell formed at conception = stem cell. Those cells go through mitosis and the earliest cells are called totipotent because they have the ability to develop into any and all types of specialized cells. Any totipotent cell has the potential to become a functioning organism. After day 4 of conception, the cells differentiate and they become pluripotent, meaning they can develop into many cell types, but not all. As they continue to differentiate, the cells become the various tissues of the body. When those cells die or are damaged, they lose the ability to perform mitosis, which makes stem cells so valuable. Adult stem cells occur in very small numbers and are very difficult to isolate. Additionally, they do not thrive in the laboratory. Stem cells could be implanted to treat degenerative disease and they can be grown in large numbers. Stem cells are derived from an embryo, which is a cause for some people to not support this research as their beliefs are that stem cells are life and the destruction is the killing of a life (fetus) in order to retrieve the stem cells. A new approach is to use mRNA to convert the patient's skin cells into stem cells. Scientists are finding that it takes about half the time of previous methods (about 17 days) and appears to be up to 100 times more effective. Additionally, the risk of rejection of the stem cells is lower. Since no embryos are killed in this method, it is preferred to the original method.

10M4=Describe the process of absorption of nutrients in the small intestine

Amino acids are absorbed via Na+ or H+ dependent cotransport. dipeptides & tripeptides are absorbed via H+ dependent cotransport. Some larger peptides are absorbed intact via transcytosis. Digestion & absorption of nutrients are completed in small intestine. Most absorbed nutrients go directly to the liver via the hepatic portal system before entering the systemic circulation. Fat digestion requires lipase & colipase. It is facilitated by bile salts that emulsify fats. Mineral absorption usually occurs via active transport. some calcium moves by the paracellular pathway. Ions & water move by the paracellular pathway as well as by membrane proteins.

2Q27Describe the general rules for the identity of neurotransmitters secreted by pre- and postganglionic neurons in the autonomic division. Be sure to specify the types of receptors, where relevant.

Anatomical differences between the branches include the fact that their nerves exit the spinal cord at different points ant their ganglia are arranged differently. The 2 branches use different neurotransmitters and receptors at synapses with target organs. The adrenal medulla is modified sympathetic ganglia. Norepinephrine release and removal at the sympathetic junction. Autonomic varicosities release neurotransmitter over the surface of the target cells. Sympathetic & parasympathetic preganglionic nurons both release acetycholine onto nicolinic cholinergic receptors. Most postganglionic sympathetic neurons secrete norepinephrine onto adrenergic receptors. Most postganglionic parasympathetic neurons secrete acetylcholine onto muscarinic cholinergic receptors. Sympathetic pathways originate in the thoracic and lumbar regions of the spinal cord. Parasympathetic pathways originate in the brainstem or the sacral region for control of pelvic organs. Sympathetic ganglia are found in ganglion chains that run along the sides of the vertebrae. Parasympathetic ganglia are located near target organs. A ganglion is a cluster of nerve cell bodies outside the CNS; equivalent to a nucleus in the CNS.Pre-ganglionic neuron originates in CNS and projects to postganglionic neuron outside CNS. Autonomic ganglia act as integrating centers.

1-14The beta cells of the pancreas produce insulin, a protein-based hormone. Which of the following organelles would be found in higher levels in the beta cells? A) None of these answers are correct B) ribosomes C) mitochondria D) lysosomes E) microvilli

B

1-28The weak interactions between atoms that keep atoms near each other but don't tightly bind them together are called A) ionic bonds. B) hydrogen bonds and van der Waals forces. C) hydrogen bonds. D) van der Waals forces and ionic bonds. E) van der Waals forces.

B

10-11The liver and skeletal muscles store glucose as ________ for a ready energy source. A) ketone bodies B) glycogen C) glucose D) lipids E) urea

B

10-14Chief cells secrete A) hydrochloric acid. B) pepsinogen. C) intrinsic factor. D) mucus. E) gastrin.

B

10-17GLUT4 transporters are A) stored in cytoplasmic vesicles. B) stored in cytoplasmic vesicles and found in adipose and skeletal muscles. C) inserted into the plasma membrane by endocytosis. D) found in adipose and skeletal muscles. E) inserted in response to glucagon.

B

10-20Before converting amino acids into intermediates for energy metabolism, they must first undergo A) denaturation. B) deamination. C) depeptidization. D) detoxification.

B

10-23Slow waves are A) cycles of smooth muscle contraction and relaxation. B) spontaneous cycles of depolarization and repolarization. C) segmental contractions. D) reflexes that originate and are integrated in the enteric nervous system. E) peristaltic contractions.

B

10-6Gluconeogenesis refers to A) removing fatty acids from adipose storage. B) creating glucose from glycerol, amino acids, or lactate. C) converting glucose to glycogen storage. D) converting glucose to storage as adipose tissue. E) removing glucose from storage as glycogen.

B

1Q11What are the two extracellular fluid compartments in the body? A) intracellular and plasma B) plasma and interstitial C) interstitial and intracellular D) plasma and the fluid portion of the blood E) None of the answers are correct.

B

1Q14If a substance has a pH that is less than 7, it is considered A) neutral. B) acidic. C) alkaline. D) a buffer. E) a salt.

B

1Q4Fick's law of diffusion states that the rate of diffusion across a membrane is A) proportional to surface area and membrane thickness, but inversely proportional to concentration gradient. B) proportional to concentration gradient, surface area, and membrane permeability. C) proportional to membrane permeability, but inversely proportional to concentration gradient and surface area. D) proportional to membrane thickness and surface area. E) All of these answers are correct.

B

2-10Which of the following statements about the Na+/K+ pump is FALSE? A) It is present in neurons. B) It transports Na+ and K+ in a 1:1 ratio. C) Its activity requires the expenditure of metabolic (cellular) energy. D) It transports Na+ out of the cell and K+ into the cell.

B

2-15The process of forming mRNA is called A) ribolation. B) transcription. C) protein synthesis. D) replication. E) translation

B

2-19Which of the following would increase the rate of diffusion across a cell membrane? A) a decrease in the surface area of the membrane B) a decrease in membrane thickness C) a decrease in the concentration gradient D) a decrease in membrane permeability

B

2-20When an enzyme is working as fast as it can because its active site is continually refilled with substrate, the condition is referred to as A) specificity. B) saturation. C) the Ricardo-Mertz phenomenon. D) allosteric modulation. E) equilibrium.

B

2-21. Activation energy is A) required to convert an endergonic reaction to an exergonic reaction. B) the energy required to bring molecules into a position where they can interact. C) the energy required for an endergonic reaction. D) the energy lost or gained in a reaction.

B

2-24Transcription occurs in the ________ of the cell. A) smooth endoplasmic reticulum B) nucleus C) cytoplasm D) plasma membrane E) Golgi apparatus

B

2-25Which of the following is a way for solutes in an aqueous solution to move from an area of high solute concentration to an area of low solute concentration? A) None of the answers are correct. B) facilitated diffusion C) active transport D) osmosis E) both facilitated diffusion and osmosis

B

2-28Which can serve as substrates for ATP production? A) glucose only B) glucose, amino acids and fatty acids C) fatty acids only D) amino acids only E) glucose and fatty acids only

B

4-18Which functions in the control of movement? A) amygdala only B) basal ganglia only C) hippocampus only D) All of the above function in the control of movement. E) hippocampus and amygdala

B

4-24Ion concentrations are first significantly affected after ________ action potential(s). A) one B) a few thousand C) a few dozen D) a few million E) a few hundred

B

4-30. In order to signal a stronger stimulus, action potentials become A) higher in amplitude and longer-lasting. B) more frequent only. C) higher in amplitude only. D) longer-lasting only. E) higher in amplitude and more frequent.

B

4-6The term axonal transport refers to A) None of the answers are correct. B) vesicle transport of proteins and organelles down the axon. C) the transport of microtubules to the axon for structural support. D) the release of neurotransmitter molecules from the axon. E) the movement of the axon terminal to synapse with a new postsynaptic cell.

B

4-9The multiple thin, branched structures on a neuron whose main function is to receive incoming signals are the A) somata. B) dendrites. C) None of the answers are correct. D) axons. E) cell bodies.

B

5-12The cell bodies of preganglionic neurons are found either in the brain stem or in the sacral region of the spinal cord. A) true for both divisions B) true only for the parasympathetic division C) true only for the sympathetic division

B

5-15The division of the autonomic nervous system that prepares the body for intense levels of activity and stress is the A) craniosacral division. B) sympathetic division. C) intramural division. D) somatomotor division. E) parasympathetic division.

B

5-21Umami is A) a flavor enhancer that is part of olfaction. B) a taste receptor cell activated by glutamate and nucleotides. C) another name for salty taste. D) the name of the ion channel activated by glutamate.

B

5-25It dominates during resting-and-digesting activities. A) true for both divisions B) true only for the parasympathetic division C) true only for the sympathetic division

B

5-9A decrease in ones perception of a stimulus whose intensity has not changed due to higher neural inhibition is A) adaptation B) habituation. C) All of these choices are correct D) divergence E) convergence

B

7-10For a diagnosis of hypertension, a patient must have A) a diastolic pressure below 80 mm Hg. B) a systolic pressure above 140 mm Hg or a diastolic pressure above 90 mm Hg. C) a systolic pressure above 140 mm Hg. D) a diastolic pressure above 90 mm Hg. E) a systolic pressure above 120 mm Hg.

B

7-28Due to the differences in opposing forces, there is net ________ occurring at the arteriolar end of most capillaries, coupled with net ________ at the venous end. A) absorption, filtration B) filtration, absorption

B

7-5The medical term for heart attack is A) fibrillation. B) myocardial infarction. C) heart block. D) heart murmur. E) heart failure.

B

2-13The tRNA molecule contains the three-base sequence called the A) aminocodon. B) anticodon. C) codon. D) semicodon.

B // A sequence of three adjacent nucleotides in tRNA designating a specific amino acid that binds to a corresponding codon in mRNA during protein synthesis; CODON=In DNA or RNA, a sequence of three nucleotides that codes for a certain amino acid or signals the termination of translation (stop or termination codon). SEE IMAGES

7-37The condition where the skin and the whites of the eyes appear slightly yellow is due to high blood levels of the substance ________.

BILIRUBIN

4M8=Describe the function of the brain stem, cerebellum, diencephalon, and cerebrum

BRAIN STEM= is oldest & most primative region of the brain.l consists of structures that derive from the embryonic midbrain & hindbrain. The nuclei in the brain stem are involved in basic processes: arousal, sleep, muscle tone, stretch reflexes, coordination of breathing, blood pressure regulation & modulation of pain. It consists of the medulla oblongata, the pons & the midbrain (In the illustration, the midbrain is on top, pons in the middle & medulla oblongata on bottom, just above the spinal cord) CEREBELLUM=2nd largest structure in the brian & located inside the base of the skull just above the base of the neck. Means little brain. Most of the nerve cells in the brain are in the cerebellum. It processes sensory info & coordinates the execution of movement. DIENCEPHALON=the "between" brain. lies btwn brain stem & cerebrum. It houses the thalamus & hypothalamus, pineal gland & pituitary gland. CEREBRUM=site for higher brain functions. It's the largest & most distinctive. fills most of the cranial cavity. It's composed of 2 hemispheres.

7M4=Explain how electrical signals are generated and propagated in the heart

Begins w/an action potential in an autorhythmic cell. The depolarization spreads rapidly to adjacent cells through gap junctions in the intercalated disks. The depolarization wave is followe by a wave of contraction that passes across the atria, then moves into the ventricles. SA node in right atrium serves as the main pacemaker of the heart. then the internodal pathway connects SA node to AV node. then to the ventricles 1) SA node fires action potential & depolarization spreads to adjacent cells thru gap junctions. 2) Rapid electrical conduction thru internodal conducting pathways 3) Slow electrical conduction thru contractile cells of atria 4) Electrical signal from AV node to AV bundle & bundle branches to apex of heart. 5) Purkinje fibers transmit impulses very rapidly so that all contractile cells in the apex contract nearly simultaneously.

10M7=Explain the lipostatic and glucostatic theories of energy balance

Blood glucose concentrations (glucostatic theory--states that glucose metabo by hypothalamic centers regulated food intake. when blood glucose concentrations decrease, the satiety center is suppressed & feeding center is dominant. When glucose metabolism increases, the satiety center inhibits the feeding center) and body fat content (the lipostatic theory--proposes that a signal from the body's fat stores to the brain modulates eating behavior so that the body maintains a particular weight. If fat stores are increased, eating decreases. In starvation, eating increases. Obesity results from disruption of this pathway.) influence food intake. It is influenced by a variety of peptides, including leptin (discovered in 1994 & means "thin". is a protein hormone & acts as a negative-feedback signal between adipose tissue & the brain. As fat stores increase, adipose cells secrete more leptin & food intake decreases), neuropeptide Y (a brain neurotransmitter that seems to be the stimulus for food intake) & ghrelin (secreted by the stomach during fasting & increases hunger when infused into human subjects).

7M10=Describe the composition of blood and the function of blood cells

Blood is the circulation portion of the extracellular fluid. The cellular elements of blood are the RBCs (erythrocytes), WBCs (leukocytes) and platelets. Function of blood cells=essential for oxygen transport and defense. Hemoglobin synthesis requires iron in the diet. Iron is transported in the blood on transferrin & stored mostly in the liver on ferritin. When hemoglobin is broken down, some heme groups are converted into bilirubin, which is incorporated into bile & excreted. Elevated bilirubin concentrations in the blood cause jaundice.

10M11=Describe the process of thermoregulation

Body temp homeostatis is controlled by the hypothalamus. Heat loss occurs from radiation, conduction, convection & evaporation. It's promoted by cutaneous vasodilation & sweating. It is generated by shivering thermogenesis & by nonshivering thermogenesis. Diet induced thermogenesis is an increase in heat production after eating. It is regulated w/i normal range (96-99 degrees F). It is homeostatically regulated.

4M(Supplement) Org of the Nervous Stm

Brain is called the seat of the soul. Nervous stm divided into 2 parts: CNS (consists of the brain & spinal cord; the PNS (peripheral nervous system) consists of sensory (afferent) neurons and efferent neurons. CNS sends output signals that travel through efferent neurons to the somatic motor division (controls skeletal muscles) and autonomic division (controls smooth and cardiac muscles, exocrine glands, some endocrine glands and some types of adipose tissue). The autonomic neurons are subdivided into sympathetic ans parasympathetic branches.

8M4=Describe the process of pulmonary ventilation

Bulk flow of air occurs down pressure gradients. Respiratory cycle= 1 inspiration + 1 expiration. Tidal volume = amt of air taken in during a single normal inspiration. Vital capacity= tidal volume PLUS expiratory & inspiratory reserve volumes. Residual Volume = air volume in the lungs at the end of maximal expiration. Air flow is directly proportional to the pressure gradient and INVERSELY related to the resistance to air flow offered by the airways. During inspiration, alveolar pressure decreases & air flows into lungs. Inspiration requires contraction of the inspiratory muscles & the diaphragm. Expiration is passive, resulting from elastic recoil of the lungs. Active expiration requires contraction of the internal intercostal and abdominal muscles. Compliance is a measure of the ease that the chest wall & lungs expand. Loss of compliance increases the work of breathing. Elastance is the ability of a stretched lung to return to its normal volume.

1-13A double covalent bond is formed when atoms A) transfer two pairs of electrons from one atom to the other. B) swap two pairs of electrons. C) share two pairs of electrons (a total of four). D) transfer a pair of electrons from one atom to the other. E) share one pair of electrons (a total of two).

C

1-21Each of the following statements concerning mitochondria is TRUE except one. Identify the exception. A) The mitochondria produce most of a cell's ATP. B) Respiratory enzymes are attached to the surface of the cristae. C) The mitochondria contain no DNA or RNA. D) The matrix of the mitochondria contains metabolic enzymes involved in energy conversion. E) The mitochondrial cristae increase the inner surface area of the organelle.

C

1-25The largest carbohydrate molecules are called polysaccharides because they are composed of ________ molecules bonded together with one another. A) pyrimidine B) nucleotide C) sugar D) amino acid E) purine

C

1-26Which of the following is NOT a function of membrane proteins? A) bind to ligands B) regulate the passage of ions C) produce energy D) anchor or stabilize the cell membrane E) act as transport molecules for various solutes

C

1Q16A placebo is A) any drug being tested in a clinical trial. B) any drug in a class of drugs commonly used as pain relievers. C) a drug or treatment that is expected to have no pharmacological effect. D) a nutritive and respiratory organ in fetal development. E) a hole in a cavity wall through which an organ protrudes.

C

1Q1Saturation occurs when A) molecules are moved by the use of vesicles. B) the energy required to move molecules results from a high-energy bond. C) a group of carrier proteins is operating at its maximum rate. D) a preference of a carrier protein for a substance is demonstrated based on the differing affinities of the carrier for the substrates. E) a carrier molecule has the ability to transport only one molecule or a group of closely related molecules.

C

1Q20All of the following are types of mediated transport except one. Identify the exception. A) facilitated diffusion B) primary active transport C) simple diffusion D) secondary active transport

C

1Q21An integrating center A) is the disturbance or change that sets the pathway in motion. B) is an electrical and/or chemical signal that travels to the effector. C) evaluates incoming signals and compares it with the setpoint. D) is the minimum stimulus that must be achieved to set the reflect response in motion.

C

2-12Aerobic metabolism of glucose A) requires oxygen. B) produces more ATP per glucose than anaerobic pathways. C) requires oxygen and produces more ATP per glucose than anaerobic pathways. D) is the fastest way to produce glucose.

C

2-18Second messenger molecules directly A) increase intracellular calcium concentration. B) change regulation of gene expression. C) All of the answers are correct. D) change enzyme activity. E) change the regulation of ion channels.

C

2-26When a chemical reaction is in equilibrium, A) the reaction has stopped. B) the reaction is proceeding at its maximum rate. C) there is no net change in the amount of substrates or products. D) there are equivalent amounts of substrates and products.

C

2-2When adenylyl cyclase is activated, A) cAMP is broken down. B) steroids are produced. C) cAMP is formed. D) protein kinases are metabolized. E) calcium ions are released from intracellular stores.

C

2Q18The resting membrane potential results from A) None of the answers are correct. B) uneven distribution of ions across the cell membrane. C) uneven distribution of ions across the cell membrane and differences in membrane permeability to Na+ and K+. D) differences in membrane permeability to Na+ and K+. E) activity of the sodium/potassium pump.

C

2Q6Which of the following would be an adequate stimulus for a chemoreceptor? A) cell stretch B) vibration C) oxygen D) cold temperature E) photon of light

C

3Q20Most of the oxygen transported by the blood is A) in ionic form as solute in the plasma. B) dissolved in plasma. C) bound to hemoglobin. D) bound to a plasma protein. E) carried by white blood cells.

C

4-11A damaged neuron has a better chance of survival and repair if the ________ is/are undamaged. A) dendrites B) Schwann cells C) cell body D) axon E) axon and dendrites

C

4-22The brain consumes about half of the ________ circulating in the body. A) potassium B) sodium C) glucose D) oxygen E) fatty acids

C

4-23The afferent and efferent axons together form the A) central nervous system. B) visceral nervous system. C) peripheral nervous system. D) somatic motor division of the nervous system. E) autonomic division system.

C

4-27Calcium is important in the synapse because it A) is necessary for acetylcholine synthesis. B) binds to receptors on the postsynaptic cell, opening ion channels, and triggering graded potentials. C) signals the exocytosis of the neurotransmitter. D) leaves the axon terminal, hyperpolarizing the cell.

C

4-31When voltage-gated K+ channels of a resting neuron open, A) K+ enters the neuron. B) K+ leaves the neuron and the neuron depolarizes. C) K+ leaves the neuron. D) the neuron depolarizes. E) K+ enters the neuron and the neuron depolarizes.

C

6-15A contraction that generates enough force to move a load is known as ________, whereas one that generates force that equals the load is known as ________. A) isotonic, eccentric B) isometric, eccentric C) isotonic, isometric D) isotropic, isometric E) isometric, isotonic

C

6-3This protein is activated by the Ca2+-calmodulin complex to phosphorylate the myosin light chain protein. A) tropomyosin B) myosin heavy chain C) myosin light chain kinase D) myosin light chain phosphatase

C

6-8Motor units that control skeletal muscles involved with fine motor movements (eye muscles or the hands) have ________ muscle fibers compared to motor units that control more gross movements (gastrocnemius muscle of the lower leg). A) the same number of B) more C) fewer

C

7-6Malnutrition can cause edema because A) interstitial protein levels are lower than plasma protein levels. B) there are not enough nutrients for plasma protein synthesis and interstitial protein levels are lower than plasma protein levels. C) there are not enough nutrients for plasma protein synthesis. D) the resulting anemia increases blood pressure. E) All of the answers are correct.

C

8-13Which of the following features of the alveolar sacs allows for the ease of diffusion of gasses? A) Elastin fibers allow the alveoli to stretch thin enough for diffusion to occur. B) They are made of a single layer of simple squamous epithelium and elastin fibers allow the alveoli to stretch thin enough for diffusion to occur. C) They are made of a single layer of simple squamous epithelium. D) Type II alveolar cells secrete surfactant.

C

8-16In the lungs, the A) blood flow rate is lower and the blood pressure is lower, respectively, than the blood flow rate and the blood pressure in other tissues. B) blood flow rate is lower and the blood pressure is higher, respectively, than the blood flow rate and the blood pressure in other tissues. C) blood flow rate is higher and the blood pressure is lower, respectively, than the blood flow rate and the blood pressure in other tissues. D) blood flow rate is higher and the blood pressure is higher, respectively, than the blood flow rate and the blood pressure in other tissues. E) blood flow rate and the blood pressure are the same as in other tissues.

C

8-19Increasing the alveolar ventilation rate will A) decrease the rate of carbon dioxide diffusion from the blood to the alveoli. B) decrease the rate of oxygen diffusion from the alveoli to the blood. C) increase the partial pressure of oxygen in the alveoli. D) have no effect on either the partial pressure or diffusion rate of gases. E) increase the partial pressure of carbon dioxide in the alveoli.

C

8-7Histamine's primary role in the respiratory system is as a A) surfactant. B) vasoconstrictor. C) bronchoconstrictor. D) bronchodilator. E) vasodilator.

C

9-15Glucose and amino acids are reabsorbed by A) diffusion. B) cotransport. C) symport with sodium. D) countertransport.

C

9-16Place the following blood vessels that carry blood to and within the kidney in the order in which blood passes through them. 1. afferent arteriole 2. efferent arteriole 3. glomerulus 4. peritubular capillary A) 4, 2, 3, 1 B) 4, 3, 2, 1 C) 1, 3, 2, 4 D) 3, 4, 2, 1 E) 1, 2, 3, 4

C

9-17Why do patients taking loop diuretics need to take supplemental potassium? A) They cause active secretion of potassium in the loop of Henle. B) They cause active reabsorption of potassium in the distal convoluted tubule. C) They inhibit the reabsorption of potassium as well as sodium in the loop of Henle. D) They inhibit intestinal absorption of potassium.

C

9-27Drugs that treat hypertension by preventing Angiotensin I from becoming Angiotensin II are called A) beta blockers. B) diuretics. C) ACE inhibitors. D) calcium channel blockers.

C

9-2All of the following will normally be found in the filtrate EXCEPT A) glucose B) urobilinogen C) erythrocytes D) potassium

C

9-3The process of filtration in the kidney is most accurately described as A) highly specific. B) completely nonspecific. C) relatively nonspecific.

C

9-4When a body is dehydrated, water in the urinary bladder A) can be returned to the circulation after moving back into the kidneys. B) can be returned to the circulation directly. C) will still be expelled from the body in the urine.

C

9-6Kidneys regulate A) both water loss and gain. B) water gain only. C) water loss only.

C

9-9In the lumen of the proximal tubule, Na+ concentration ________ the Na+ concentration inside the cells of the tubule wall. A) is about the same as B) is much lower than C) is much higher than D) is slightly lower than E) is slightly higher than

C

2-30Match the type of transport with its description. A. transcellular transport B. absorption C. paracellular transport D. secretion 1) in one side of a cell and out the other 2) from an organ's lumen to the extracellular fluid 3) between adjacent cells 4) movement from the extracellular fluid into the lumen of an organ

C paracellular transport: 3between adjacent cells, B absorption: 2from an organ's lumen to the extracellular fluid, A transcellular transport: 1in one side of a cell and out the other, D secretion: 4movement from the extracellular fluid into the lumen of an organ

4-1Match these terms with their descriptions. A. propriospinal tracts B. gray matter C. descending tracts D. white matter E. ascending tracts 1. projections that carry sensory information to the brain 2. unmyelinated, consists of cell bodies, dendrites, and axon terminals 3. projections of white matter that remain in the spinal cord 4. carry primarily efferent signals from the brain 5. myelinated axons with very few cell bodies

C=4 descending tracts:carry primarily efferent signals from the brain, A=3 propriospinal tracts:projections of white matter that remain in the spinal cord, B=2 gray matter:unmyelinated, consists of cell bodies, dendrites, and axon terminals, E=1 ascending tracts:projections that carry sensory information to the brain, D=5 white matter:myelinated axons with very few cell bodies

3Q33Too little oxygen in cells is often accompanied by too much ________, which is called ________.

CARBON DIOXIDE, HYPERCAPNIA

1M4=List the four types of macromolecules that make up living things, and be able to describe the chemical structure of each

CARBS=Most abundant biomolecule. Name means Carbon with Water. They are monosaccharides, disaccharides & polysaccharides. Most carbs in body are polysach's. The chitin in bugs, glycogen, cellulose in plants, starch, yeasts & bacteria make dextran. LIPIDS=biomolecules made mosty of C & H (glycerol) Non-polar & not very soluble in water. Glycerol +3 fatty acids = triglyceride (90% of lipids = triglycerides). Also include eicosanoids, steroids & phospholipids. PROTEINS=polymers of smaller building-block molecules called amino acids. All have a Carboxyl Group, an Amino Group & a Hydrogen plus a variable R group.There are 20. Assemble into polymers called peptides (covalent bond btwn 2 AA's). NUCLEOTIDES=includes DNA, RNA, ATP & cyclic AMP. Have a phosphate, a 5-C sugar & C-N ring (it's base) called a nitrogenous base. THEY ALL CONTAIN HYDROGEN, CARBON & OXYGEN.

10M2=Describe the process and locations of digestion of carbohydrates, proteins, and fats

CARBS=most are disaccharides 7 complex carbs. cellulose is not digestible. Must be digested to monosaccharides before absorbed in the small intestine (in the lumen). PROTEINS=chains of amino acids. Endopeptidases include pepsin in the stomach & trypsin and chymotrypsin in the small intestine. endopeptidase digests internal peptide bonds. Exopeptidases digest terminal peptide bonds to release amino acids. After digestion, proteins are absorbed mostly as free amino acids. Egg protein is the most digestible. LIPIDS=most are hydrophobic & must be emulsified to facilitate digestion in the aqueous enviro of the intestine. Bile salts facilitate fat digestion. Digestion & absorption is in the small intestine. Form fat droplets called micelles. Occurs primarily by simple diffusion. cholesterol is avtively transported. chylomicrons are absorbed into the lymph. Fat-soluble vitamins are absorbed along w/fats. Water soluble vitamins are absorbed by mediated transport. Vit B12 requires intrinsic factor secreted by the stomach.

3Q31Plasma consists of blood minus ________ and ________.

CELLS, PLATELETS

6-30Rhythmic reflexive motor activities, such as breathing or walking, are controlled by networks of neurons in the central nervous system called

CENTRAL PATTERN GENERATORS

10M3=Explain the processes of chemical and mechanical digestion

CEPHALIC PHASE: Chemical & Mechanical digestion begins in the mouth. Saliva starts it (can be triggered by multiple stimuli=sight, smell, touch & thought of food). Chemical digestion begins w/secretion of salivary amylase. It breaks starch into maltose after the enzyme is activated by Cl- in saliva. Mechanical begins in the oral cavity w/chewing. Lips, tongue, teeth all contribute to mastication creating a softened, moistened mass (bolus) that can be easily swallowed. Swallowing is a reflex integrated by a medullary center. Motility is the mechanical portion of moving food from the mouth to the anus & the mixing of food. Enzymatic breakdown of food = chemical. GASTRIC PHASE: Stomach stores food, begins protein & fat digestion & protects body from swallowed pathogens. Secretes mucus & bicarbonate from mucous cells, gastric acid from parietal cells , pepsinogen from chief cells, somatostatin from D cells, histamine from ECL cells 7 gastrin from G cells. Gastric fcn is integrated w/the cephalic & intestinal phases of digestion. INTESTINAL PHASE: Acid in the intestine, CCK & secretin delay gastric emptying. Pancreas secretes bicarbonate to neutralize gastric acid. Intestinal enzymes are part of the brush border. Most pancreatic enzymes are secreted as zymogens that need to be activated. Most digestion takes place in small intestine. Digestion & absorption of nutrients are completed in small intestine. Most absorbed nutrients go directly to the liver via the hepatic portal system before entering the systemic circulation. the large intestine concentrates the 1.5 liters of chyme that enter it daily. Undigested material in the colon moves forward by mass movement. The defecation reflex, a spinal reflex subject to higher control, is triggered by sudden distension of the rectum. Colonic bacteria use fermentation to digest organic material. Cells of the colon can both absorb & secrete fluid. Excessive fluid secretion or decreased absorption causes diarrhea (that can cause dehydration).

10-42After processing in the stomach, the gastric contents are referred to as ________.

CHYME

6M7=Explain the differences between fast- and slow-twitch muscles

CLASSIFIED BY SPEED AND FATIGUE RESISTANCE. FAST-TWITCH: large diameter, pale color, easily fatigued. (labeled W for White muscle). SLOW-TWITCH: smaller diameter, darker color due to myoglobin, fatigue resistant. Has numerous mitochondria & extensive capillary blood supply.

3Q49Write the chemical equation catalyzed by the enzyme carbonic anhydrase. Suppose the concentration of H+ is increased by an outside force in a solution that had been at equilibrium. According to the law of mass action, what must happen to the CO2 concentration to reestablish equilibrium after this disturbance? What must happen to the concentration of bicarbonate?

CO2 + H2O ↔ H2CO3 ↔ H+ + HCO3- Carbon dioxide concentration will increase as the reaction is shifted to the left. Bicarbonate concentration will decrease.

8M10=Explain how carbon dioxide influences systemic pH

CO2 is the primary stimulus for changes in ventilation. O2 & plasma pH play lesser roles. If too little O2 is in the blood, ventilation increase. If the rate of CO2 production by the cells exceeds the rate of CO2 removal by the lungs, arterial PCO2 increases & ventilation is intensified to match CO2 removal to production. If arterial PCO2 raises above the threshold level for the chemoreceptors, the receptors fire 7 the control network increases ventilation.

7-30The osmotic pressure created by the presence of proteins is known as ________, which is ____________ (higher/lower) in the plasma than in the interstitial fluid.

COLLOID OSMOTIC OR ONCOTIC PRESSURE; HIGHER

6-23 1. Briefly compare and contrast the structure and behavior of cardiac, smooth, and skeletal muscles.

Cardiac: found only in the heart and moves blood through the circulatory system. It is classified as striated because of their alternating light and dark bands seen under the light microscope. Involuntary muscles. Skeletal: are attached to the bones of the skeleton, enabling these muscles to control body movement. They are classified as striated because of their alternating light and dark bands seen under the light microscope. Mostly defined as voluntary muscles. Smooth: primary muscle of internal organs and tubes such as the stomach, urinary bladder and blood vessels. Its primary function is to influence the movement of material into, out of and within the body. Viewed under the microscope, smooth muscle lacks the obvious cross-bands of striated muscles. The lack of banding results from the less organized arrangement of contractile fibers within the muscle cells. Involuntary muscles.

4M1=Describe the basic anatomical features of neurons

Carry electrical signals rapidly & over long distances. They have long, thin extensions or processes, that can extend up to a meter in length. In most pathways, neurons release chemical signals called neurotransmitters into the extracellular fluid to como w/neighboring cells. Some are linked by gap junctions allowing electrical signals to pass directly from cell to cell.

1M8=List the function of the cellular organelles

Cell membrane separates the inside enviro of the cell (intracellular fluid) from the extracellular fluid. Inside, the cell is divided into the nucleus & cytoplasm. (CYTOPLASM consists of: the cytosol (fluid portion); membrane bound structures called organelles (mitochondria (site of most ATP synthesis in cell), endoplasmic reticulum (the rough ER is main site of protein syntheis; smooth ER=main site of lipid synthesis), golgi apparatus (protein modification & packaging), lysosomes (storage; digestive stm of cell), perixosomes), insoluble particles called inclusions (lipid droplets, glycogen granules & ribosomes (manufacture protein under the direction of DNA)); and protein fibers that create the cytoskeleton (provides strength, support and internal org, links cells together).

8M5=List factors influencing diffusion at the alveoli

Changes in alveolar surface area (loss of surface area = emphysema) , in diffusion barrier thickness (scar tissue = fibrotic lung diseases), and in fluid distance between the alveoli & pulmonary capillaries (pulmonary edema) can all affect gas exchange efficiency & arterial PO2. Diffusion problems cause hypoxia. Gas solubility affects diffusion= 1) the pressure gradient of the gas; 2) the solubility of the gas in liquid; 3) temperature.

8M6=Describe common respiratory diseases

Changes in alveolar surface area (loss of surface area/destruction of alveoli= emphysema) , in diffusion barrier thickness (scar tissue, thickened alveolar membrane slows gas exchange. loss of lung compliance may decrease alveolar ventilation = fibrotic lung diseases), and in fluid distance between the alveoli & pulmonary capillaries (fluid in interstitial space increase diffusion distance. Arterial PCO2 may be normal due to higher CO2 solubility in water. pulmonary edema) can all affect gas exchange efficiency & arterial PO2. Asthma=increased airway resistance decreases alveolar ventialtion. Bronchioles constricted. hypoxia==too little oxygen in blood hypercapnia==too much CO2 in blood (these 2 are clinical signs, not diseases) Diffusion problems cause hypoxia. Gas solubility affects diffusion= 1) the pressure gradient of the gas; 2) the solubility of the gas in liquid; 3) temperature.

4M3=Explain the role of chemical signaling in the nervous system

Chemical factors alter electrical activity. a large variety of chemicals alter the conduction of action potentials by binding to Na+, K or Ca2+ channels in the nuuron membrane. EX: some neurotoxins bind tand block Na+ channels (local anesthetics like procaine). If Na+ channels are not functional, Na+ cannot enter the axon. Changes in blood K+ concentration affect resting membrane potential and the conduction of action potentials.

3M4=Describe the hypothalamic-pituitary axis

Classic endocrine cells act as both sensor & integrating center in the simple reflex pathway. Many endocrine reflexes involve the nervous system, either through neurohormones or through neurons that influence hormone release. The pituitary gland is composed of the anterior pituitary (a true endocrine gland) and the posterior pituitary (an extension of the brain). The posterior pituitary releases 2 neurohormones, oxytocin & vasopressin that're made in the hypothalamus. Hypothalamic releasing hormones & inhibition hormones control the secretion of anterior pituitary hormones. In complex endocrine reflexes, hormones of the pathway act as negative feedback signals.

1-17Homeostasis is the ability of the body to A) prevent excessive blood loss. B) prevent the external environment from changing. C) ignore external stimuli to remain in a state of rest. D) quickly restore changed conditions to normal. E) prevent the internal environment from changing

D

1-18The esophagus is a tube that carries food from the mouth to the stomach. It does not secrete any enzymes or absorb any nutrients, but it does need to stand up to significant friction and stress. The type of epithelium most likely lining the esophagus would be: A) cuboidal epithelium. B) simple squamous epithelium. C) simple columnar epithelium. D) stratified squamous epithelium. E) transitional epithelium.

D

1-1In regard to lipids, the term unsaturated refers to A) the ring structure of steroids. B) the lack of double bonds between adjacent carbon atoms in a fatty acid. C) glycerol, which acts as an anchor for joined fatty acids. D) the presence of double bonds between adjacent carbon atoms in a fatty acid. E) fats, such as butter and lard, which come from animal sources.

D

1-22Most of the fat found in the human body is in the form of A) monoglycerides. B) prostaglandins. C) phospholipids. D) triglycerides. E) steroids.

D

1-2You conduct an experiment on twenty 18-year-old male subjects to see how various intensities of exercise influence heart rate. Which of the following is/are considered an independent variable? A) sex of subjects B) More than one of the answers is correct. C) heart rate D) intensity of exercise E) age of subjects

D

1-5When an enzyme reaches its saturation point, A) the amount of substrate for the enzyme to act upon is very low and the amount of product produced by the enzyme decreases. B) the amount of substrate for the enzyme to act upon is very low. C) the amount of product produced continues to increase. D) the amount of substrate for the enzyme to act upon is very high. E) the amount of product produced by the enzyme decreases.

D

1-7Which of the following is a buffer zone between the outside world and most of the cells of the body? A) Blood B) All of the answers are correct C) Lumen D) Extracellular fluid E) Lymph

D

10-22Brown fat A) contains a rich vascular supply. B) is innervated by the sympathetic nervous system. C) functions in nonshivering thermogenesis. D) All of these answers are correct. E) is found in infants.

D

10-25A harmful substance that the liver produces when metabolizing fatty acids is A) ammonia. B) beta units. C) rancid fatty acids. D) ketone bodies. E) urea.

D

10-27During the fasting state, the energy stores of the ________ become the major source of glucose for the whole body. A) brain B) adipose tissues C) muscles D) liver E) pancreas

D

10-2A bomb calorimeter measures A) both carbon dioxide produced and oxygen consumed when a food sample is completely combusted. B) the oxygen consumed when a food sample is completely combusted. C) the carbon dioxide produced when a food sample is completely combusted. D) the heat produced when a food sample is completely combusted. E) only the food calories available to the human body.

D

10-9In the digestive system, HCl is released by ________, whereas HCO3- is secreted primarily from the ________. A) the pancreas, parietal cells of the stomach B) the liver, parietal cells of the stomach C) parietal cells of the stomach, liver D) parietal cells of the stomach, pancreas

D

1Q13Essential amino acids that are used to build proteins A) exist in twenty six different configurations. B) are linked together by ionic chemical bonds in proteins. C) can be used medically for both diagnosis and treatment of diseases. D) can be derived from the foods we eat and digest. E) can only be made by cells within our bodies.

D

1Q18The most important energy-transferring compound in cells is a nucleotide known as A) glucose. B) fructose. C) protein. D) adenosine triphosphate. E) deoxyribonucleic acid.

D

1Q3Which of the following elements form polar covalent bonds? A) carbon and hydrogen B) carbon and carbon C) sodium and chlorine D) hydrogen and oxygen E) hydrogen and hydrogen

D

1Q5Cholesterol is A) a precursor to steroid hormones. B) a structural component of cell membranes. C) a dangerous fat that is absent from a healthy body. D) a precursor to steroid hormones and a structural component of cell membranes

D

1Q9You conduct an experiment on twenty 18-year-old male subjects to see how various intensities of exercise influence heart rate. Which of the following is/are considered a dependent variable? A) age of subjects B) sex of subjects C) intensity of exercise D) heart rate E) More than one of the answers is correct.

D

2-11Cyclic AMP activates A) protein hormones. B) a G protein. C) adenylyl cyclase. D) protein kinase A. E) hormone receptors.

D

2-14The ________ of glycogen from many glucose molecules is a ________ reaction. A) decomposition, endergonic B) decomposition, exergonic C) synthesis, exergonic D) synthesis, endergonic

D

2-16Which is NOT a basic method of cell-to-cell communication? A) cytoplasmic transfer of signals B) nerve and blood-transported signals C) diffused chemical signals D) mechanical signals E) contact-dependent signals

D

2-17Bulk flow is fluid flow as a result of a(n) ________ gradient. A) electrical B) None of the answers are correct. C) concentration D) pressure

D

2-5Which of the following statements about the resting membrane potential is TRUE? A) It is due in part to the presence of extracellular proteins. B) The inside of the membrane is positively charged compared to the outside. C) It is normally equal to zero volts. D) It results, in part, from the concentration gradients for Na+ and K+

D

2-9The reactions of glycolysis occur in the cell's ________; the reactions of the citric acid cycle occur in the ________. A) mitochondria, cytoplasm B) cytoplasm, cytoplasm C) mitochondria, mitochondria D) cytoplasm, mitochondria

D

2Q20The loudness or intensity of a sound wave is related to its A) frequency. B) duration. C) pitch D) amplitude.

D

2Q22When voltage-gated Na+ channels of a resting neuron open, A) Na+ enters the neuron. B) the neuron depolarizes. C) Na+ leaves the neuron and the neuron depolarizes D) Na+ enters the neuron and the neuron depolarizes. E) Na+ leaves the neuron.

D

3Q22The main function of hemoglobin is to A) aid in the process of blood clotting. B) All of the answers are correct. C) protect the body against infectious agents. D) carry oxygen. E) carry nutrients from the intestine to the body's cells.

D

3Q7Each myosin head has a binding site for A) actin only. B) calcium and ATP. C) ATP only. D) actin and ATP. E) calcium only.

D

3Q8If a person is bedridden for several days, the baroreceptor reflex may fail upon standing. Why? A) Blood pools in the feet and legs and there is reduced blood flow to the brain. B) Blood pools in the feet and legs. C) There is reduced blood flow to the brain. D) The kidneys have reduced the blood volume. E) All of the statements are correct.

D

4-28Which of the following will best increase the conduction rate of action potentials? A) Decrease the diameter of the axon, decrease the resistance of the axon membrane to ion leakage. B) Decrease the diameter of the axon, increase the resistance of the axon membrane to ion leakage. C) All of the answers above are correct. D) Increase the diameter of the axon, increase the resistance of the axon membrane to ion leakage. E) Increase the diameter of the axon, decrease the resistance of the axon membrane to ion leakage.

D

4-8. An excitatory postsynaptic potential (EPSP) A) hyperpolarizes a neuron, increasing the likelihood of an action potential. B) None of these answers are correct. C) depolarizes a neuron, decreasing the likelihood of an action potential. D) depolarizes a neuron, increasing the likelihood of an action potential. E) depolarizes a neuron, decreasing the likelihood of an action potential.

D

5-18Nicotine enhances the release of ________ in the brain. A) serotonin B) epinephrine C) acetylcholine D) dopamine E) glutamate

D

5-19The pitch of a sound wave is related to its A) amplitude. B) duration. C) decibels. D) frequency.

D

5-22With the exception of olfaction, all sensory pathways first travel to the ________, which acts as a relay and processing station. A) hypothalamus B) cerebellum C) cerebrum D) thalamus E) medulla oblongata

D

5-29The gate control theory of pain modulation states that pain transmission can be blocked by A) stimulation of C-fibers. B) tonic activity of the inhibitory neurons in the dorsal horn. C) cold stimulation of the A-delta fibers. D) mechanical stimulation of A-beta fibers.

D

6-16The function of transverse tubules is to A) store Ca2+ ions inside the muscle fiber. B) conduct ATP molecules out of the mitochondria throughout the sarcoplasm. C) ensure a supply of glycogen throughout the muscle sarcoplasm. D) rapidly conduct action potentials to the interior of the muscle fiber.

D

6-5The brief period of time between the beginning of the action potential in the muscle and the beginning of contraction is referred to as the A) repolarization period. B) relaxation phase. C) refractory period. D) latent period. E) depolarization period.

D

6-6For antagonistic muscle groups to move a limb, flexor contraction occurs coincident with A) contraction of the extensor. B) no changes in the extensor. C) contraction of the tendon. D) relaxation of the extensor.

D

6-7The effector in a reflex is the A) sensory receptor. B) control center. C) afferent neuron. D) muscle or gland. E) efferent neuron.

D

8-12Harry suffers from cystic fibrosis and frequently has periods where he can hardly breathe. The problem is the result of A) collapse of one or both lungs B) inflammation of the bronchi. C) constriction of the trachea. D) thick secretions that exceed the ability of the mucus elevator to transport them. E) laryngospasms that occur in response to a toxic substance produced by the epithelial cells.

D

8-15The process by which dissolved gases are exchanged between the blood and interstitial fluids is A) pulmonary ventilation. B) external respiration. C) breathing. D) diffusion. E) cellular respiration.

D

8-2An increase in PCO2 would cause A) the bronchioles to constrict and the systemic arterioles to dilate. B) None of the answers are correct. C) the bronchioles to dilate and the systemic arterioles to constrict. D) the bronchioles to dilate and the systemic arterioles to dilate. E) the bronchioles to constrict and the systemic arterioles to constrict.

D

9-21The force for glomerular filtration is the A) ATP-dependent processes in the nephron. B) None of the answers are correct. C) fluid pressure produced by the displacement of the fluid in the lumen of the tubules. D) blood pressure in the glomerular capillaries. E) osmotic pressure in the glomerular capillaries.

D

9-8Atrial natriuretic peptideA) B) C) D) E) A) inhibits the release of renin. B) increases the GFR and stimulates the release of renin. C) increases the GFR. D) increases the GFR and inhibits the release of renin. E) stimulates the release of renin.

D

1-27The watery medium that surrounds a cell is known as A) cytoplasm. B) plasma. C) protoplasm. D) extracellular fluid. E) cytosol.

D // CYTOPLASM=thick liquid residing between the cell membrane holding organelles, doesn't contain the nucleus; PLASMA=colorless fluid part of blood/lymph/milk where corpuscles or fat globules are suspended; PROTOPLASM=colorless material comprising the living part of a cell, including the cytoplasm, nucleus, and other organelles. Protoplasm is the living contents of a cell that is surrounded by a plasma membrane. It is a general term for the cytoplasm; CYTOSOL= the site where many chemical reactions take place. In prokaryotes, it is where most metabolic reactions take place. In eukaryotes, it is where the organelles and other cytoplasmic structures are suspended (AKA intracellular fluid or cytoplasmic matrix) is the liquid found inside cells. It is the liquid of a cell, that is parted from other parts of the cell by cell walls, such as the mitochondrial matrix inside the mitochondrion.

10-35 ________ is the amount of heat generated during the digestion of a meal.

DIET-INDUCED THERMOGENESIS

6-31The T-tubular membrane contains ________ receptors that are ________ and therefore respond to action potentials. This receptor is mechanically linked to ________ receptors in the adjacent sarcoplasmic reticulum.

DIHYDROPYRIDINE (DHP) OR L-TYPE CALCIUM CHANNEL, VOLTAGE-SENSING, RYANODINE (RyR) OR CALCIUM RELEASE CHANNEL

2Q38________ is a decrease in the number of receptors of target cells in order to dampen the effects of excess hormone.

DOWN-REGULATION

10-41The three sections of the small intestine, in order according to movement of its contents, are ________, ________, and ________.

DUODENUM, JEJUNUM, ILEUM

10M1=List the organs of the GI tract and their respective functions

Digestive System begins with: Oral Cavity (mouth & pharynx) = receptacle for food. Moves to GI Tract: Esophagus=food travels through thorax to abdomen.\ Stomach=storage center that holds food & makes chyme. (The pyloric valve empties food into the small intestine) Small Intestine= includes the duodenum, jejunum & ileum. Digestion is essentially completed in the small intestine & nearly all digested nutrients & secreted fluids are absorbed there. (Accessory organs = pancreas & liver. Sphincter of Oddie keeps pancreatic fluid & bile from entering the small intestine except during a meal.) the rest of the chyme goes to large intestine. Large Intestine = water & electrolytes. Colon=proximal section of large intestine where chyme is converted into semisolid feces as water/electrolytes are absorbed out of the chyme & into the ECF. Rectum=where feces leaves.

1-12Which of the following elements combine to form nonpolar covalent bonds? A) hydrogen and oxygen B) nitrogen and hydrogen C) carbon and chlorine D) sodium and chlorine E) carbon and hydrogen

E

1-6Tay-Sachs Disease is caused by a mutation that affects the function of A) rough endoplasmic reticulum. B) mitochondria. C) Golgi bodies. D) smooth endoplasmic reticulum. E) lysosomes.

E

1-9Which of the following substances is most alkaline? A) stomach secretions, pH = 1 B) lemon juice, pH = 2 C) white wine, pH = 3 D) tomato juice, pH = 4 E) urine, pH = 6

E

10-12The beta cells of the pancreas produce A) glucagon. B) digestive enzymes. C) renin. D) cortisol. E) insulin.

E

10-16Mucus functions in A) protection only. B) protection, lubrication, and enzyme activation. C) lubrication only. D) enzyme activation only. E) protection and lubrication.

E

10-19The nutrients that yield the most energy per gram when metabolized are A) carbohydrates. B) proteins. C) nucleic acids. D) vitamins. E) fats.

E

10-3The brain can use ________ for energy. A) only lactate B) only fats C) only ketones D) only glucose E) both glucose and ketones

E

1Q2Which type of tissue below has minimal extracellular matrix? A) epithelial only B) connective only C) neural only D) muscle only E) epithelial, neural, and muscle

E

1Q6Which of the following are considered differences between endocrine and neural control systems? A) specificity B) nature of the signal C) speed D) duration of action E) All of the answers are correct.

E

1Q8Nucleic acids are polymers of units called A) amino acids. B) fatty acids. C) bases. D) ribose. E) nucleotides.

E

2-4Receptor molecules are located A) in the nucleus. B) in the cytosol. C) in the outer cell membrane. D) on the cell surface. E) in all of these places.

E

2Q15The pituitary hormone that stimulates milk production by the mammary glands is A) growth hormone. B) FSH C) TSH. D) ACTH E) prolactin

E

2Q17An area of the retina that contains only cones and is the site of sharpest vision is the A) outer segment. B) optic disc. C) inner segment. D) tapetum lucidum. E) fovea.

E

2Q2The brain and spinal cord together compose the A) visceral nervous system. B) peripheral nervous system. C) somatic motor division of the nervous system. D) autonomic division system. E) central nervous system.

E

2Q3 An important structure in both learning and memory is the A) hypothalamus. B) cerebellum. C) pons. D) medulla. E) hippocampus.

E

2Q5The pituitary hormone that controls hormone synthesis and release from the thyroid gland is A) STH B) LH C) FSH D) ACTH E) TSH

E

2Q9Which ion(s) is/are higher in concentration inside the cell compared to outside? A) More than one of the answers is correct. B) calcium C) sodium D) chloride E) potassium

E

3Q1Striated muscles are so-called because of a repeating pattern of light and dark bands. One repeating unit of the banding pattern is called a A) myofilament. B) myomere. C) sarcofibril. D) crossbridge. E) sarcomere.

E

3Q21Put the following in order: 1. End-plate potentials trigger action potentials. 2. Transverse tubules convey potentials into the interior of the cell. 3. Acetylcholine binds to receptors on the motor end plate. 4. Ca2+ is released from the sarcoplasmic reticulum. A) 4, 3, 2, 1 B) 3, 1, 4, 2 C) 1, 2, 3, 4 D) 2, 1, 4, 3 E) 3, 1, 2, 4

E

3Q29Type I alveolar cells A) are phagocytic. B) allow rapid diffusion of gases through their thin membranes, secrete a chemical known as surfactant, and are phagocytic. C) None of the statements are true. D) secrete a chemical known as surfactant. E) allow rapid diffusion of gases through their thin membranes.

E

3Q2The actual sites of gas exchange within the lungs are A) bronchioles. B) terminal sacs. C) pleural spaces. D) alveolar ducts. E) alveoli.

E

3Q3Edema is likely to occur when A) blood hydraulic pressure at the capillary decreases. B) hemorrhage occurs. C) the concentration of protein in the blood increases. D) the blood hydraulic pressure in a capillary is equal to the blood osmotic pressure. E) the heart becomes an insufficient pump.

E

3Q6 1. right ventricle 2. left atrium 3. right atrium 4. pulmonary artery 5. left ventricle 6. pulmonary vein A) 2, 5, 6, 4, 3, 1 B) 3, 2, 4, 6, 1, 5 C) 4, 2, 5, 6, 3, 1 D) 1, 3, 6, 4, 5, 2 E) 3, 1, 4, 6, 2, 5

E

4-10Exocrine glands, smooth muscles, and cardiac muscles are controlled by the A) central nervous system. B) enteric nervous system. C) peripheral nervous system. D) somatic motor division. E) autonomic nervous system.

E

4-12Which type of synapse is most prevalent in the nervous system? A) processing B) radiative C) mechanical D) electrical E) chemical

E

4-15. The sodium-potassium exchange pump A) moves sodium and potassium in the direction of their chemical gradients. B) transports sodium ions into the cell during depolarization. C) transports potassium ions out of the cell during repolarization. D) must re-establish ion concentrations after each action potential. E) requires ATP to function

E

4-16During the relative refractory period, an initial threshold-level depolarization is usually not sufficient to trigger an action potential. Why? A) The statement is incorrect; a threshold-level depolarization always triggers an action potential. B) Only K+ channels are still open, so Na+ entry is offset by K+ loss. C) Only a few K+ channels have returned to their resting position. D) Only some Na+ channels have returned to their resting position. E) Some Na+ channels have returned to their resting position and K+ channels are still open, so Na+ entry is offset by K+ loss.

E

4-26Myelin is formed by A) axons B) oligodendrocytes C) all of the above answers are correct D) Schwann cells E) Schwann cells and oligodendrocytes

E

4-4Which of the following is the most common location where action potentials originate? A) synaptic bouton B) dendrites C) synaptic cleft D) cell body E) axon hillock

E

4-7The rising phase of the action potential is due to A) Na+ flow out of the cell and K+ flow into the cell. B) K+ flow into the cell only. C) Na+ flow out of the cell only. D) K+ flow out of the cell only. E) Na+ flow into the cell only.

E

5-14Sound waves are converted into mechanical movements (vibrations) by the A) cochlea. B) round window. C) oval window. D) malleus. E) tympanic membrane.

E

5-23Antagonistic control of efferent output is typical of the ________ division. A) somatic B) somatic and sensory C) somatic and autonomic D) sensory E) autonomic

E

5-4The primary purpose of the middle ear bony structures (maleus, incus and stapes) is to A) dampen the vibration as it conducts to the cochlea. B) equalize pressure in the middle ear. C) transduce the sound waves into vibration. D) amplify the vibration as it conducts to the cochlea. E) transmit otitis media to the cochlea.

E

5-5A blind spot in the retina occurs where A) amacrine cells are located. B) ganglion cells synapse with bipolar cells. C) the fovea is located. D) rod cells are clustered to form the macula. E) the optic nerve leaves the eye.

E

7-16At an intercalated disc, A) the cell membranes of two cardiac muscle fibers are completely separated by a synapse. B) All of the answers are correct. C) t-tubules unite the membranes of the adjoining cells. D) the myofibrils are loosely attached to the membrane of the disc. E) two cardiac muscle cells are connected by gap junctions.

E

7-20The usual treatment for neonatal jaundice is exposing the infant to certain wavelengths of light. The infant wears only a diaper and undergoes the light treatment continuously for several days, even at night. Why can the infant's treatement then be discontinued? A) All of the answers are correct. B) because the light generates additional RBCs C) because the digestive tract is then large enough to secrete bile D) because light treatment stimulates liver growth E) because the transition between fetal hemoglobin to adult is complete

E

7-21How would a decrease in the concentration of iron in the blood affect the process of hemostasis? A) Coagulation would proceed more rapidly. B) Platelet plugs would fail to form. C) Retraction would occur prematurely. D) Coagulation would proceed more slowly. E) There would be no effect.

E

7-3The vessels that are the main site of variable resistance in the circulatory system, and that contribute more than 60% of the total resistance, are the A) elastic arteries. B) muscular arteries. C) venules. D) veins. E) arterioles.

E

7-4The action potential in a cardiac contractile cell causes A) opening of HCN channels. B) activation of NCX transporters. C) opening of ryanodine receptor calcium channels. D) activation of sodium-potassium ATPase. E) opening of L-type calcium channels.

E

7-9Reactive hyperemia is A) None of the answers are correct. B) lack of blood flow due to an allergic reaction. C) increased blood pressure after stress. D) reflex contraction of smooth muscle in response to stress. E) increased blood flow following a period of reduced blood flow.

E

8-18The distance between the alveolar airspace and capillary endothelium is ________, allowing gases to diffuse ________ between them. A) None of the answers are correct-distance does not affect diffusion rate. B) long, rapidly C) long, slowly D) short, slowly E) short, rapidly

E

8-1A typical value for intrapleural pressure is ________ mm Hg. A) -6 B) 0 C) +6 D) +3 E) -3

E

8-4Place the following structures of the respiratory tree in the order in which air passes through them. 1. secondary bronchi 2. bronchioles 3. primary bronchi 4. alveoli 5. terminal bronchioles A) 1, 3, 5, 2, 4 B) 4, 1, 2, 3, 5 C) 1, 3, 2, 5, 4 D) 3, 1, 5, 2, 4 E) 3, 1, 2, 5, 4

E

8-5Type II alveolar cells A) are phagocytic. B) allow rapid diffusion of gases through their thin membranes. C) allow rapid diffusion of gases through their thin membranes, secrete a chemical known as surfactant, and are phagocytic. D) None of the statements are true E) secrete a chemical known as surfactant.

E

8-8Which is NOT considered to be a primary function of the respiratory system? A) vocalization B) regulation of pH balance C) protection against pathogens D) gas exchange E) regulation of water balance

E

9-10As a result of respiratory alkalosis, A) the tidal volume increases. B) the kidneys conserve bicarbonate. C) the respiratory rate increases. D) the kidneys secrete fewer hydrogen ions. E) the body retains less carbon dioxide

E

9-18Angiotensin II A) stimulates thirst and causes widespread vasoconstriction throughout the body. B) stimulates thirst only. C) causes the synthesis and release of aldosterone from the adrenal cortex. D) causes widespread vasoconstriction throughout the body only. E) stimulates thirst, causes widespread vasoconstriction throughout the body, and causes the synthesis and release of aldosterone from the adrenal cortex.

E

9-20Stimuli for the activation of the RAS pathway include A) low blood pressure in arterioles in the nephron only. B) high blood pressure in the renal artery only. C) a decrease in fluid flow through the distal tubule only. D) low blood pressure in arterioles in the nephron, a decrease in fluid flow through the distal tubule, and high blood pressure in the renal artery. E) low blood pressure in arterioles in the nephron and a decrease in fluid flow through the distal tubule.

E

9-22Urine is carried to the urinary bladder by A) blood vessels. B) the urethra. C) lymphatics. D) All of the answers are correct E) the ureters.

E

9-26Blood flow through the kidney includes a feature seen in only a few organs. What is it? A) arterial shunts B) veins containing highly oxygenated blood C) anastomoses D) vascular sinuses E) portal system

E

3Q37A tracing of the electrical activity of the heart, monitored by electrodes placed on the skin, is called a(n) ________.

ELECTROCARDIOGRAM or ECG

3Q34________ is characterized by a decreased surface area for gas exchange in the lungs.

EMPHYSEMA

8-24In the disease ________, many symptoms are due to destruction of elastic fibers in the lung.

EMPHYSEMA

1M9=List characteristics of the four tissue types, and use information about a tissue to determine what type it is

EPITHELIAL=protect the internal enviro, reg the exchange of material or manufacture/secrete chemicals. 5 functional types: exchange (gases (holes in cells)), transporting (non-gaseous btwn internal/external enviro), ciliated (move fluid/particles across tissue surface (microvilli), protective (prevent exchange btwn internal/external enviro (skin)) & secretory (release secretory products into external enviro or blood (mucus or hormones into blood)). CONNECTIVE=provides structural support & forms a physical barrier (loose are under the skin; dense are tendons, ligaments (made of collagen), adipose stores fat, blood is a watery matrix; cartilage is solid & flexible w/o blood supply, bone is deposits of calcium salts) MUSCLE=propogate action potentials. cardiac, smooth & skeletal NEURAL=includes neurons (use electrical & chemical signals to transmit info to different parts of body) and glial cells (support cells for neurons).

3Q30The hormone that regulates the production of red blood cells is ________.

ERYTHROPOIETIN

1Q32The means by which a cell transports large molecules out of the cell is called

EXOCYTOSIS

1M5=Describe and apply knowledge of the chemical bonding properties of carbon, hydrogen, nitrogen, and oxygen

Electrons are impt for covalent & ionic bonds, energy capture & transfer & formation of free radicals. Covalent bonds=form when adjacent atoms share 1 or more electrons. Ionic bonds=strong & formed when oppositely charged ions are attracted to each other. Hydrogen bonds= weak. form when H atoms in polar molecues are attraced to O, N or Fl atoms. Hydrogen bonding is responsible for the surface tension of water. Van der Waals forces=weak. form when atoms are attracted to each other. Polar molecules=atoms share electrons unevenly. NON-Polar=atoms share electrons evenly. ION-atom that gains or loses electrons acquires an electrical charge

9M3=Describe the processes of filtration, reabsorption, secretion, and excretion

FILTRATION: Allows most components of plasma to enter the tubule but excludes blood cells & most plasma proteins. Capillary pressure causes filtration. GFR is relatively constant & is subject to autoregulation. REABSORPTION: Most reabsorption takes place in proximal tubule. Fine regulation takes place in the distal segments. Active transport of Na+ & other solutes creates concentration gradients for passive reabsorption of urea & other solutes. Glucose, amino acids, ions & various organic metabolites are reabsorbed by Na+-linked 2nd-ary active transport. Most renal transport is mediated by membrane proteins & exhibits saturation, specificity & competition. The renal threshold is the plasma concentration when a substance 1st appears in the urine. Peritubular capillaries reabsorb fluid along their entire length. Can be active or passive. SECRETION: Enhances excretion by removing solutes from the peritubular capillaries. K+, H+ & a variety of organic compounds are secreted. Molecules that compete for renal carriers slow the secretion of a molecule. EXCRETION: Excretion rate depends on 1) its filtered load; 2) whether it is reabsorbed or secreted as it passes thru the nephron. Clearance=how many mm of plasma passing thru kidneys have been totally cleared of a solute in a given period of time. Can be used to determine how the nephron handles a solute filtered into it. Inulin clearance is = to GFR. Creatine is used to measure GFR. MICTURATION: pissing

6M3=List the steps involved in contraction and relaxation of skeletal and smooth muscle

FORCE CREATED BY A CONTRACTING MUSCLE IS CALLED MUSCLE TENSION. THE LOAD IS A WT OR FORCE THAT OPPOSES CONTRACTION OF A MUSCLE. SMOOTH= 1) Intracellular Ca2+ concentrations increase when Ca2+ enters cell & is released from sarcoplasmic reticulum. 2) Ca2+ binds to calmodulin 3) Ca2+-calmodulin activates myosin light chain kinase (MLCK). 4) MLCK phosphorylates light chains in myosin heads & increases myosin ATPase activity 5) Active myosin cross bridges slide along actin & create muscle tension. 6) Free Ca2+ in cytosol decreases when Ca2+ is pumped out of the cell or back into the sarcoplasmic reticulum. 7) Ca2+ unbinds from calmodulin. MLCK activity decreases. 8) Myosin phosphatase removes phosphate from myosin light chains, which decreases myosin ATPase activity 9) Less myosin ATPase activity results in decreased muscle tension. SKELETAL= 1) Somatic motor neuron releases ACh at neuromuscular junction. 2) Net entry of Na+ through ACh receptor-channel initiates a muscle action potential. 3) Action potential in t-tubule alters conformation of DHP receptor. 4) DHP receptor opens RyR Ca2+ release channels in sarcoplasmic reticulum, and Ca2+ enters cytoplasm. 5) Ca2+ binds to troponin, allowing actin-myosin binding. 6) Myosin heads execute power stroke. 7) Actin filament slides toward center of sarcomere. 8) Sarcoplasmic Ca2+ -ATPase pumps Ca2+ back into SR. 9) Decrease in free cytosolic causes Ca2+ to unbind from troponin. 10) Tropomyosin recovers binding site. When myosin heads release, elastic elements pull filaments back to their relaxed position. SMOOTH MUSCLE CONTRACTION/RELAXATION ARE SIMILAR TO SKELETAL MUSCLE BUT DIFFER: 1) Ca2+ COMES FROM THE ECF AS WELL AS THE SARCOPLASMIC RETICULUM; 2) AN ACTION POTENTIAL IS NOT REQUIRED FOR Ca2+ RELEASE; 3) THERE IS NO TROPONIN, SO Ca2+ INITIATES CONTRACTION THROUGH A CASCADE THAT INCLUDES PHOSPHORYLATION OF MYOSI LIGHT CHAINS; 4) AN ADDITIONAL STEP IN SMOOTH MUSCLE RELAXATION IS DEPHOSPHORYLATION OF MYOSIN LIGHT CHAINS BY MYOSIN PHOSPHATASE.

6M5=Understand how reciprocal inhibition of flexor-extensor pairs is essential to skeletal muscle function

For muscle contraction to extend the leg, the antagonistic flexor muscles must relax, called reciprocal inhibition. In the leg, this requires relaxation of the hamstring muscles running up the back of the thigh.

1M7=Explain the role of enzymes in cell metabolism

Functional groups of proteins include enzymes that speed up chemical reactions, signal molecules and the receptor proteins that bind signal molecules and specialized proteins that function as biological pumps, filters, motors or transporters. they are biological catalysts that speed up chemical reactions. They play an important role in metabolism.

2M7=distinguish between different types of signaling receptors

G proteins linked to amplifier enzymes are the most prevalent signal transduction system. G-protein coupled receptors alter ion channels. The G protein-coupled adenylyl cyclase-cAMP protein kinase A pathway is the most common pathway for protein and peptide hormones. Ligand-gated ion channels open or close to create electrical signals. Response of a cell to a signal is determined by the cell's receptor for the signal. receptors = isoforms. AGONISTS mimic the action of a signal molecule. ANTAGONISTS block the signal pathway. Receptor proteins exhibit specificity, competition & saturation. Too much saturation, the cells will use down-regulation (decreases # of receptors). Or up-regulation (increase # of receptors).

9-29The amount of filtrate entering the proximal tubules of the kidneys each minute is the ________.

GFR OR GLOMERULAR FILTRATION RATE

9-35 2. Describe GFR autoregulation, and explain why it is important.

GFR is glomerular filtration rate and it is the volume of fluid that filters into Bowman's capsule per unit time. Average GFR is 125 ml/min or 180 L/day. This means that the kidneys filter the entire plasma volume 60 times/day or 2.5 times every hour. If most of the filtrate were not reabsorbed during its passage through the nephron, we would run out of plasma in only 24 minutes of filtration. It is influenced by 2 factors: the net filtration pressure and the filtration coefficient. Filtration pressure is determined primarily by renal blood flow and blood pressure. The filtration coefficient has 2 components: the surface area of the glomerular capillaries available for filtration and the permeability of interface between the capillary and Bowman's capsule. In this respect, glomerular filtration is similar to gas exchange at the alveoli, where the rate of gas exchange depends on partial pressure differences, the surface are of the alveoli and the permeability of the alveolar-capillary diffusion barrier. Blood pressure provides the hydrostatic pressure that drives glomerular filtration. You would think that it would follow the path of BP in that when BP goes up or down, then GFR would go up or down, but that is rarely the case. GFR is relatively constant. As long as mean arterial blood pressure remains between 80 mm Hg - 180 mm Hg, GFR averages 180 L/day. It is controlled primarily by regulation of blood flow through the renal arterioles. If the overall resistance of the renal arterioles increase, blood flow decreases and blood is diverted to other organs. The effect of increased resistance on GFR depends on where the resistance takes place. If resistance is at the afferent arteriole, then hydrostatic pressure decreases on the glomerular side. If resistance is at the efferent arteriole, then hydrostatic pressure increases on the glomerular side. Increased glomerular pressure increases GFR. Most regulation occurs at the afferent arteriole. Autoregulation of glomerular filtration rate is a local control process in which the kidney maintains a relatively constant GFR in the face of normal fluctuations in blood pressure. One important function of GFR autoregulation is to protect the filtration barriers from high blood pressures that might damage them. We do not completely understand the autoregulation process, but several mechanisms are at work. The myogenic response is the intrinsic ability of vascular smooth muscle to respond to pressure changes. Tubuloglomerular feedback is a paracrine signaling mechanism through which changes in fluid flow through the loop of Henle influence GFR. Hormones and autonomic neurons also influence GFR.

10-32The pancreatic hormone that raises blood glucose concentration is ________.

GLUCAGON

10-40The process of synthesizing glucose from lipids, amino acids, or other carbohydrates is called ________.

GLUCONEOGENESIS

9-33The excretion of glucose in the urine is called ________.

GLUCOSURIA OR GLYCOSURIA

10-37The process of glycogen formation is known as ________.

GLYCOGENESIS

9M5=List the factors influencing GFR

Glomerular filtration rate (GFR) is the amount of fluid that filters into Bowman's capsule per unit time. Average GFR is 125 mL/min or 180 L/day. Hydrostatic pressure in glomerular capillaries can be altered by changing resistance in the afferent & efferent arterioles. Autoregulation of glomerular filtration is accomplished by a myogenic response of vascular smooth muscle in response to pressure changes & by tubuloglomerular feedback. When fluid flow thru the distal tubule increases, the macula densa cells send a paracrine signal to the afferent arteriole, which constricts. Reflex control of GFR is mediated thru systemic signals, such as hormones & thru the autonomic nervous system.

2M2=explain the basics of how ATP is produced and utilized as an energy carrier in cells

Glycolysis takes place in the cytosol of the cell, but the CAC is in the mitochondria, requiring txport of substrates across the mito membrane. It produces 2 pyruvate that yields 2 atp, 2 nadh & 2 H+. Does not require oxygen. Aerobic metabolism of pyruvate through the CAC yields ATP CO2 & high energy electrons captured by NADH & FADH2. The high energy electrons from NADH & FADH2 give up their electrons as they pass through the electron txport system. Max energy yield for 1 glucose = 30-32 ATP. ANAEROBIC metabo=pyruvate converted to lactate w/yield of 2 ATP for each glucose.

2Q26Explain the two reasons why graded potentials lose strength as they move through the cell. Why don't action potentials lose strength?

Graded potentials are changes in membrane potential whose magnitude is proportional to the stimulus and that decreases with distance as it spreads through the cytoplasm. 1) They degrade due to current leak--allow positive charge to leak out into the extracellular fluid. 2) Cytoplasmic resistance--resistance to the flow of electricity. Action potentials are spikes or electrical signals of uniform strength that travel form a neuron's trigger zone to the end of it's axon. Ion channels in the axon membrane open sequentially as electrical current passes down the axon. Action potentials do not lose strength because Additional Na+ entering the cell reinforce the depolarization.

10-30Match each product with the cell or region that secretes or contains it. A. HCl B. mucus C. enzymes D. HCO3- E. more than one of these pancreatic cells goblet cells parietal cells brush border

HCl:parietal cells, mucus:goblet cells, enzymes:brush border, more than one of these:pancreatic cells

3-16The trophic hormones reach the pituitary through the

HYPOTHALAMIC-HYPOPHYSEAL PORTAL SYSTEM

3Q39Too little oxygen in cells is called ________.

HYPOXIA

7M1=List the components of the circulatory system

Heart, blood vessels & blood = known as the cardiovascular system. Blood vessels away from heart = arteries Blood vessels return blood to heart = veins. Septum=the central wall that divides the heart. Atrium= receives blood returning to the heart from the blood vessels Ventricle=pumps blood out of the blood vessels It is a series of tubes (blood vessels) filled w/fluid (blood) and connected to a pump (heart). Pressure generated in the heart propels blood through the system continuously. Its primary fcn is to transport materials t/from all parts of the body: 1) nutrients, water & gases that enter from the external enviro 2) materials that move from cell to cell w/i the body; 3) wastes that the cells eliminate.

2Q28Carl is suffering from hypothyroidism. What is hypothyroidism? How could his physician determine whether the malfunction is in the hypothalamus and pituitary or at the level of the gland itself?

Hypothyroidism is a condition in which the thyroid gland does not make enough thyroid hormone. Perform blood tests: TSH. This hormone is made in the pituitary gland. It is released into the bloodstream. It stimulates the thyroid gland to make thyroxine. If the level of thyroxine in the blood is low, then the pituitary releases more TSH to try to stimulate the thyroid gland to make more thyroxine. Therefore, a raised level of TSH means the thyroid gland is underactive and is not making enough thyroxine. Thyroxine (T4). A low level of T4 confirms hypothyroidism.

2Q40Graded potentials that arrive at postsynaptic neurons are called ________ if they make that cell less likely to fire.

INHIBITORY

9-34 2. "Glucose, glucose, every where, nor any speck to utilize" is a phrase made up by one of the authors working on this test question. Similar to the irony of not being able to prevent dehydration by drinking seawater, people with untreated diabetes mellitus are unable to prevent starvation despite the large amount of glucose surrounding their cells; as if that isn't bad enough, dehydration is also a problem. Explain why there is glucose in the urine of such people, why glucose is not present in the urine of normal people, and why diabetics become dehydrated.

If renal transport of glucose reaches saturation, then all available carriers are occupied by substrate. With glucose at saturation, it has reached the transport maximum. At normal plasma glucose concentrations, all glucose that enters the nephron is reabsorbed before it reaches the end of the proximal tubule. The tubule epithelium is well supplies with carriers to capture glucose as the filtrate flows past. When blood glucose concentrations become excessive, glucose is filtered faster than the carriers can reabsorb it. The carriers become saturated and are unable to reabsorb all the glucose that flows through the tubule. As a result, some glucose escapes reabsorption and is excreted in the urine. Excretion of glucose in the urine is called glucosuria or glycosuria and usually indicates an elevated blood glucose concentration. Diabetics become dehydrated because they are using the fluid to try to remove all of the excess glucose, but they are unable to. Normally, the kidneys try to make up for high glucose levels in the blood by allowing the extra glucose to leave the body in the urine. If you do not drink enough fluids, or you drink fluids that contain sugar, the kidneys can no longer get rid of the extra glucose. Glucose levels in the blood can become very high as a result. The blood then becomes much more concentrated than normal (hyperosmolarity). Hyperosmolarity is a condition in which the blood has a high concentration of salt (sodium), glucose, and other substances that normally cause water to move into the bloodstream. This draws the water out of the body's other organs, including the brain. Hyperosmolarity creates a cycle of increasing blood glucose levels and dehydration.

3-11 2. What normally stops hormone action, and why is this important?

If the signal activity by hormones and other chemical signals are not limited, the system becomes unable to function properly and it is potentially fatal (Silverthorne, 2013, p. 211). When it comes to insulin secretion in the body, it is terminated in three ways: limiting insulin secretion, removing/inactivating insulin circulating in the blood or by terminating insulin activity in target cells (Silverthorne, 2013, p. 211). The hormones in the blood are degraded into inactive metabolites by enzymes (primarily found in the liver and kidneys); then, excreted in either the bile or urine. The rate of hormone breakdown is called it's "half life". Hormones that are bound to target membrane receptors can be broken down in several ways: enzymes in the plasma can degrade peptide hormones bound to cell membrane receptors; receptor-hormone complex is brought to the cell by endocytosis and then the hormone is digested in the lysosomes; intracellular enzymes metabolize hormones that enter cells. Additionally, feedback loops can stop hormone action. Examples of feedback loops: negative, positive, short-loop negative feedback loops and long-loop negative feedback loops. Whether it is enzyme action or feedback loops, it is important to stop hormone action so that the body does not over or under compensate. This could lead to fatigue or death of the organ, system or even host.

3Q47Millie's grandfather suffers from congestive heart failure. Whenever she visits him, she notices that his ankles and feet appear to be quite swollen. She knows that you are an avid student of anatomy and physiology, and she asks you why this occurs. What would you tell her?

In heart failure, the heart is not able to produce enough force to circulate the blood properly. The blood tends to pool in the extremities and as more and more fluid accumulates in the capillaries, the blood hydraulic pressure increases. This results in a fluid shift from the blood to the interstitial space. The fluid accumulation exceeds the ability of the lymphatics to drain it and as a result, edema occurs and produces the obvious swelling.

2-36 3. Aquaporins are cell membrane water channels. Why does the cell need water channels when the plasma membrane is permeable to water anyway?

In order to answer why the cell would need aquaporins when the plasma membrane is permeable to water, we need to understand the make-up of water and the cell membrane. The water molecule is polar with its H+ and OH- ions. Cell membranes are selectively permeable. Whether or not a substance enters a cell depends on the properties of the cell membrane and those of the substance. The lipid and protein composition of a given cell membrane determines which molecules will enter the cell and which will leave. Membrane permeability is variable and can be changed by altering the proteins or lipids of the membrane. The water is attracted to the cell membrane by its polar charge, but water cannot diffuse through the lipid bilayer because it cannot penetrate through the non-polar fatty acid tail of the lipid bilayer. The aquaporin, a special water channel, is necessary because it helps to facilitate the movement of water through the cell membrane. The aquaporin plays a vital role in osmotic equilibrium. The movement of water across a membrane in response to a solute concentration gradient is called osmosis and the pressure that the water faces in moving from one compartment to another that opposes the osmotic movement is called osmotic pressure.

5-34 1. Describe the major anatomical differences between the sympathetic and parasympathetic branches.

In short: *Sympathetic pathways originate in the thoracic and lumbar (middle) regions of the spinal cord. *Parasympathetic pathways originate in the brain stem or the sacral region for control of pelvic organs. *Sympathetic ganglia are found in ganglion chains that run along the sides of the vertebrae *Parasympathetic ganglia are located near target organs. ***** Sympathetic ganglia are found primarily in two ganglion chains that run along either side of the bony vertebral column, with additional ganglia along the descending aorta. Long nerves (axons of postganglionic neurons) project from the ganglia to the target tissues. Because most sympathetic ganglia lie close to the spinal cord, sympathetic pathways generally have short preganglionic neurons and long postganglionic neurons. Many parasympathetic pathways originate in the brain stem, and their axons leave the brain in several cranial nerves. Other parasympathetic pathways originate in the sacral region (near the lower end of the spinal cord) and control pelvic organs. In general, parasympathetic ganglia are located either on or near their target organs. Consequently, parasympathetic preganglionic neurons have long axons and parasympathetic postganglionic neurons have short axons.

2-37 3. Briefly explain the effects of the hormone insulin on glucose transport. What is diabetes mellitus? How are Types I and II different? Why do insulin injections help alleviate one type but not the other?

Insulin is an anabolic hormone because it promotes glycogen, protein and fat synthesis. When insulin is absent or deficient, cells go into catabolic metabolism. Additionally, it: 1) increases glucose transport into most, but not all, insulin-sensitive cells; 2) it enhances cellular utilization and storage of glucose; 3) it enhances utilization of amino acids; 4) it promotes fat synthesis. Diabetes mellitus is a condition characterized by hyperglycemia resulting from the body's inability to use blood glucose for energy. It is a family of metabolic disorders caused by defects in the homeostatic pathways that regulate glucose metabolism. Several forms exist and some can be inherited. In Type 1 diabetes, the pancreas no longer makes insulin and therefore blood glucose cannot enter the cells to be used for energy. The endocrine cells of the pancreas are destroyed and stop making insulin, a protein hormone involved in blood glucose homeostasis. In Type 2 diabetes, either the pancreas does not make enough insulin or the body is unable to use insulin correctly. The insulin is present in normal or above-normal levels, but the insulin-sensitive cells of the body do not respond normally to the hormone. If people with type 1 diabetes are given shots of insulin, their blood glucose levels decline. If people with type 2 diabetes are given insulin, blood glucose levels may change very little. In type 1, insulin levels are low; therefore, it is more likely to cause upregulation of the insulin receptors. Glucose is lipophobic so it cannot cross the cell membrane by simple diffusion. It must go by facilitated diffusion. If a cell lacks the necessary carriers, facilitated diffusion cannot take place—which is why glucose cannot diffuse into the cells when the blood glucose concentration is higher than the intracellular glucose concentration.

7M8=Explain the function of the lymphatic system

It interacts w/3 other systems: cardiovascular system, digestive system & immune system. they return fluid & proteins filtered out of the capillaries to the circulatory system; 2) pick up fat absorbed at the small intestine & transferring it to the circulatory system; 3) serve as a filter to help capture & destroy foreign pathogens.. It allows the 1-2ay movement of interstitial fluid from the tissues into the circulation. Lymph capillaries lie close to all blood capillaries except those in the kidney & CNS.

1Q27Your chemistry professor probably stressed to you that catalysts don't make chemical reactions happen; they only speed up chemical reactions that would happen anyway. But in the body, the presence or absence of active enzymes is the primary determinant of whether a metabolic pathway will be active or not. Is this a contradiction? Explain why or why not.

It is not a contradiction. Since enzymes speed up chemical reactions, without them a process that would normally take a fraction of a second could take hours, months or years. Theoretically, yes, the reaction would occur, but at a length of time that would cause other issues within the cell/body that could lead to cell death. Enzymes control the metabolism, which regulates the movement of substrate from one cellular compartment to another.

2Q31In multiple sclerosis, there is progressive and intermittent damage to the myelin sheath of central axons. One symptom is poor motor control of the affected area. Why does destruction of the myelin sheath affect motor control?

It's actually 2 different tissue types that merged during embryonic development. The anterior pituitary is a true endocrine gland of epithelial origin, derived from embryonic tissue that formed the roof to the mouth. it's also called the adenohypophysis and its hormones are adenohypophyseal secretions. The posterior pituitary, or neurohypophysis is an extension of the neural tissue of the brain. It secretes neurohormones made in the hypothalamus, a region that controls many homeostatic functions.

2Q32How does the embryological origin of the two distinct parts of the pituitary gland relate to the control of secretion and the types of hormones produced?

It's actually 2 different tissue types that merged during embryonic development. The anterior pituitary is a true endocrine gland of epithelial origin, derived from embryonic tissue that formed the roof to the mouth. it's also called the adenohypophysis and its hormones are adenohypophyseal secretions. The posterior pituitary, or neurohypophysis is an extension of the neural tissue of the brain. It secretes neurohormones made in the hypothalamus, a region that controls many homeostatic functions.

3Q36The condition where the skin and the whites of the eyes appear slightly yellow is called ________.

JAUNDICE

7-38The turbulent flow of blood causes a noise called a ________ that can be heard through the stethoscope when taking blood pressure.

KOROTKOFF SOUND

10-31Bile is produced in the ________ and stored in the ________.

LIVER, GALLBLADDER

10-43 2. A condition known as lactose intolerance is characterized by painful abdominal cramping, gas, and diarrhea. The cause of the problem is an inability to digest the milk sugar, lactose. How would this cause the observed symptoms? Why is yogurt less likely to trigger the symptoms?

Lactose intolerance causes cramping, gas and diarrhea because the bacteria in the large intestine ferment lactose to gas and organic acids, leading to bloating and flatulence (gas) that can also lead to cramping. Decreased lactase activity is associated with a condition known as lactose intolerance so if a person drinking milk or eats diary products, diarrhea may result. The simplest way to remedy this is to remove milk products from the diet; however, there are products that have predigested lactase available. Yogurt is less likely to trigger the symptoms because the friendly bacteria in yogurt have done much of the digesting of the lactase for you.

1M(Supplement) Protein interactions

Ligands bind to proteins at binding site. Induced-fit model, shapes of the ligand and binding site do not have to match exactly. Affinity-attraction of a protein to its ligans. Agonists-compete for a ligand's binding site. Some proteins must be activated by proteolytic activation or by adding cofactors. Competitive inhibitors can be displaced from the binding site, but irreversible antagonists cannot. Allosteric modulators bind to proteins at a location other than the binding site. Covalent modulators bind w/covalent bonds. Both types of modulators may activate or inhibit the protein. Extreme temp or pH will denature proteins. The amt of protein directly influences the magnitude of the cell's response (done by up/down regulation). If the amt of protein (like an enzyme) is constant, the amt of ligand determines the cell's response. If all binding proteins (like enzymes) become saturated with ligand, the response reaches its max.

7M9=Describe the process of hematopoiesis and be able to discuss disorders of hematopoiesis

Means "blood" & "formation". It is the synthesis of blood cells, begins early in embryonic development & continues throughout a person's life. As the embryo develops, blood cell production spreads from the yolk to the liver, spleen & bone marrow. From birth to age 5, hematopoiesis is only in the marrow of the bones of the skeleton. By adult years, it's only in the pelvis, spine, ribs, cranium & proximal ends of long bones. Disorders: Disorders of hematopoiesis underlie a number of hematologic malignancies and other disorders such as leukemia, aplastic anemia, lymphoma, myelodysplasia, myeloproliferative disorders, and inborn errors of metabolism. Chemotherapy-induced cytopenia is one of the primary causes of morbidity and mortality in the treatment of cancer.

7M7=Describe how blood pressure is measured and explain why blood pressure is important

Measured with a sphygmometer. blood squeezing through a compressed brachial artery makes Korotkoff sounds. High BP increases the risk of heart disease & stroke, the 1st & 3rd leading causes of death for Americans. Can also cause kidney disease & blindness.

10M8=Explain how the human body can utilize stored energy in the fasted state

Metabolism is all the chemical reactions that extract, use or store energy. anabolic pathways synthesize small molecules into larger ones. Catabolic pathways break large molecules into smaller ones. The function of fasted-state metabolism is to maintain adequate plasma glucose concentrations bcz glucose is normally the only fuel that the brain can metabolize. Glycogenolysis is glycogen breakdown. Gluconeogenesis is glucose synthesis from noncarbohydrated precursors, especially amino acids. In the fasted state, the liver produces glucose from glycogen & amino acids. Beta oxidation (CAC) of fatty acids forms acidic ketone bodies. When there are not enough carbs, then the fats are broken down into ketone bodies. The brain can use glucose & ketones for fuel.

3-15Chemical signals released into the blood by neurons are called

NEUROHORMONES

6-24 Name the components of a reflex pathway, and describe generally how a reflex works.

Name the components of a reflex pathway, and describe generally how a reflex works. Reflex pathways in the nervous system consist of chains or networks of neurons that link sensory receptors to muscles or glands. They can be classified in several ways: 1) Efferent division that controls the effector a) somatic motor neurons control skeletal muscles b) autonomic neurons control smooth and cardiac muscles, glands and adipose tissue 2) Integrating region within the CNS a) spinal reflexes do not require input from the brain b) cranial reflexes are integrated within the brain 3) Time at which the reflex develops a) innate (inborn) reflexes are genetically determined b) learned (conditioned) reflexes are acquired through experience 4) The number of neurons in the reflex pathway a) monosynaptic reflexes have only 2 neurons: 1 afferent (sensory) and 1 efferent. Only somatic motor reflexes can be monosynaptic b) polysynaptic reflexes include 1 or more interneurons between the afferent and efferent neurons. All autonomic reflexes are polysynaptic because they have 3 neurons: 1 afferent and 2 efferent

3Q46One of the side effects of prolonged bedrest is loss of blood volume. Explain why this is this case and the role of baroreceptors in this phenomenon.

Normally, blood pressure is slightly lower in the upper part of the body than in the lower part of the body due to gravity. The main baroreceptors are in the aorta and carotid artery. During prolonged bedrest, blood will be spread more evenly in the lower and upper parts of the body, and thus, the baroreceptors will perceive increased blood pressure. They will induce compensatory mechanisms that lower blood volume.

1M2=Describe common experimental models used in the study of physiology

Observation & experimentation are the key elements of scientific inquiry. Hypothesis=logical guess. Independent variable=factor manipulated by the investigatior. Dependent variable=observed factor. Well designed studies have a control to ensure changes are from the study & not outside forces. Data=prepared as a graph. Longitudinal studies=carried out over long period of time. Prospective Study=study healthy people for years to ID factors that contrib to cardiovascular disease.(Helps kids & grandkids of the studied group) Cross-sectional=study a pop. for prevalence of disease (i.e., age group or socioeconomic factor=cause of disease). Retrospective=match people w/a specific disease to a group of similar people who are healthy.

4M9=Explain the organization of the cerebral cortex

Organized into functional areas: serves an an integrating ctr for sensory info & decision-making region. 1)SENSORY receive sensory input & translate it into perception (awareness). 2)MOTOR direct skeletal muscle movement 3)ASSOCIATION integrate info from sensory & motor areas & can direct voluntary behaviors.

8M8=Explain the role of hemoglobin in gas transport and factors that influence hemoglobin binding

Oxygen is transported dissolved in plasma & bound to hemoglobin. Oxygen hemoglobin binding is influenced by pH, temp & 2,3-diphosphoglycerate (2,3-DPG). The PO2 of plasma determines how much O2 binds to hemoglobin.

2Q42The term that describes the need for one hormone to be present for a second hormone to produce a full effect is called ________.

PERMISSIVE

7-34In hemostasis, vasoconstriction is rapidly followed by mechanical blockage of the hole by a ________.

PLATELET PLUG

8-29The opposing layers of pleural membrane are held together by a thin film of ________.

PLEURAL FLUID

3Q41A ________ reflex has at least one interneuron placed between the sensory and motor neuron.

POLYSYNAPTIC

3-12A circulatory route with two capillary beds in series is known as a

PORTAL SYSTEM

6-28Asynchronous activation of motor units is important because it functions to

PREVENT FATIGUE

6-29_________ monitor the position of skeletal muscles and joints

PROPRIOCEPTORS

7-35The ________ circuit carries blood to and from the alveoli of the lungs.

PULMONARY

3Q43In ________, fluid accumulates in the interstitial spaces of the lungs, slowing gas exchange

PULMONARY EDEMA

3M3=List major endocrine glands and the hormones they produce

Pineal=melatonin Hypothalamus=trophic (CONTROL THE SECRETION OF OTHER HORMONES) Posterior Pituitary=oxytocin, vasopressin Anterior Pituitary=prolactin, growth hormone (somatotropin), follicle-stimulating hormone, lutenizing hormone, thyroid-stimulating hormone & adrenocorticotrophic hormone Thyroid Gland=Calcitonin, triiodothyronin & thyroxine Parathyroid Gland=parathyroid hormone Thymus Gland=Thymosin, Thymopoietin Pancreas=Insulin, glucagon, somatostatin, pancreatic polypeptide Adrenal Cortex=Aldosterone, cortisol, androgens Testes=Androgens, inhibin Ovaries=estrogen, progesterone, inhibin, relaxin (pregnancy)

7-41 2. Blood clotting is considered to be an example of a positive feedback situation. Explain why this is so, and demonstrate the appropriate steps or areas of hemostasis as part of your explanation.

Platelets adhere to collagen with the help of integrins, membrane receptor proteins that are linked to the cytoskeleton. Binding activates platelets so that they release the contents of their intracellular granules, including serotonin, ADP and platelet-activating factor (PAF). PAF sets up a positive feedback loop by activating more platelets. It also initiates pathways that convert platelet membrane phospholipids into thromboxane A2. Serotonin and thromboxane A2 are vasoconstrictors. They also contribute to platelet aggregation, along with ADP and PAF. The net result is a growing platelet plug that seals the damaged vessel wall. If platelet aggregation is a positive feedback event, what prevents the platelet plug from continuing to form and spreadin gbeyond the site of injury to other areas of the vessel wall? The fact that platelets do not adhere to normal endothelium. Intact vascular endothelial cells convert their membrane lipids into prostacylin, and eicosanoid that blocks platelet adhesion and aggregations. Nitric oxide released by normal, intact endothelium also inhibits platelets from adhering. The combination of platelet attraction to the injury site and repulsion from the normal vessel wall creates a localized response that limits the platelet plug to the area of damage.

7M11=Explain the process of hemostasis

Platelets are cell fragments filled w/granules containing clotting proteins & cytokines. They are activated by damage to vascular endothelium. Hemostasis begins w/vasoconstriction and the formation of a platelet plug. Exposed collagen triggers platelet adhesion &U platelet aggregation. The plug is converted into a clot when reinforced by fibrin. Then, fibrin is made from fibrinogen through the action of thrombin. As the vessel is repaired, plasmin trapped in the platelet plug dissolves fibrin and breaks down the clot. Platelet plugs are restricted to the site of injury by prostacyclin in the membrane of intact vascular endothelium. anticoagulants limit the extent of blood clotting w/i a vessel.

8-30Premature babies frequently need to be put on a respirator to help them breathe. Why does this become a necessary treatment for many premature babies?

Premature babies often have lung problems because the lungs are the last of the organs to develop. Respiratory disease is the most frequent illness/complication of premies. One such disease, respiratory distress syndrome is a breathing disorder that most often affects infants borne 6 weeks or more before their due dates. The infant's lungs are not able to make enough surfactant that keeps the lungs open so that they can breathe in air once they are born. Without enough surfactant, the lungs collapse and the infant has to work hard to breathe. He or she might not be able to breathe in enough oxygen to support the body's organs. Infants may also develop bronchopulmonary dysplasia (BPD) where they have fewer healthy air sacs and tiny blood vessels in their lungs. Both air sacs and tiny blood vessels that support them are needed to breathe. Another reason why a baby may need to be put on a respirator is that the diaphragm is not strong enough to support breathing. In premature babies, the muscles (including the lungs and diaphragm) may not have enough oxygen to bring to the cells to provide the energy and resources needed for proper muscle function like breathing.

4-33The period of time during which an excitable membrane can respond again, but only if the stimulus is greater than the initial stimulus is the

RELATIVE REFRACTORY PERIOD

5M4=Describe the anatomy of the eye and the mechanism of perception of visual stimuli

RETINA=has photoreceptors that transduce light energy into and electrical signal that passes to the visual cortex for processing PUPIL=changes its size depending on amt of light hitting it CILIARY MUSCLE=contracts/relaxes based on light waves focused by the lens PHOTORECEPTORS=converts light into electrial energy and is located on the retina. Signals pass through the bipolar neurons to ganglion cells, whose axons form the optic nerve. FOVEA=HAS MOST ACUTE VISION BCZ IT HAS THE SMALLEST RECEPTIVE FIELDS. Rods=nighttime vision CONES=high-acuity vision & color vision in daylight

9M1=List the functions of the kidney

Regulate extracellular fluid volume; blood pressure & osmolarity; maintain ion balance; regulate pH; excrete wastes & foreign substances; participate in the endocrine pathways.

1M3=Describe challenges associated with physiology research

Results difficult to interpret bcz: 1) variability (tremendous genetic/enviro variability); 2) psychological factors (placebo-a sugar pill that causes people to think they have the real thing & experience healing; nocebo-tell a group of the side effects & they'll get them; blind study-the subjects do not know if they got tx or placebo; double blind-study--3rd party not involved in the study are only ones who know if the pt. got the tx or placebo; double-blind crossover study--ctl group in 1st 1/2 becomes experimental group in the 2nd 1/2); 3) ethical considerations 9using humans as experiments. Meta-analysis=combines data from many studies to look for trends.

6M1=Explain differences between skeletal, smooth, and cardiac muscle cells

SKELETAL=attached to bones of skeleton enabling the muscles to control body movement. Voluntary muscles.Skeletal & cardiac muscles are striated (grooved--have light/dark bands). controlled by somatic motor neurons. SMOOTH=primary muscle of internal organs & tubes such as the stomach, bladder & blood vessels. Primary fcn is to influence the movement of material into, out of and w/i the body. Like food through the GI tract. Smooth muscles are involuntary. controlled by autonomic innervation, paracrines & hormones. CARDIAC=found only in the heart and moves blood through the circulatory system. Skeletal & cardiac muscles are striated (grooved--have light/dark bands). involuntary muscle.controlled by autonomic innervation, paracrines & hormones. electrically linked through gap junctions.

6M4=Be able to describe neural control of smooth and skeletal muscle contraction

SKELETAL=controlled by the CNS bcz somatic motor neurons always cause contraction in skeletal muscle. Contraction in smooth and cardiac muscles may occur spontaneously or may be controlled by hormones or byt he autonomic division of the nervous system.

6-34____________ reflexes involve skeletal muscles.

SOMATIC

5M3=Distinguish between somatic and special senses

SOMATIC MOTOR DIVISION CONTROL SKELETAL MUSCLES. IT ORIGINATES IN THE CNS & TERMINATES ONA SKELETAL MUSCLES. THEY ARE ALWASY EXCITATORY & CAUSE MUSCLE CONTRACTIONS. SPECIAL SENSES: vision, hearing, taste, smell & equilibrium. SOMATIC SENSES: touch, temperature, pain, itch & proprioception

4-34The addition of several stimuli arriving from different locations on the same cell to produce an action potential is called

SPATIAL SUMMATION

2-31The ability of a carrier molecule to transport only one specific molecule or a group of closely related molecules is called _______.

SPECIFICITY

10-38The primary complex carbohydrate ingested and digested by people is ________.

STARCH

6-25Muscle spindles are _____________ receptors and cause reflex____________, whereas Golgi tendon organs respond primarily to __________ and cause a _____________ reflex.

STRETCH, CONTRACTION, TENSION, RELAXATION

3Q44The ________ circuit carries blood to and from all parts of the body except the alveoli of the lungs.

SYSTEMIC

9M6=Explain the role of ADH, aldosterone, angiotensin II, and ANP in regulation of salt and water balance

Salt appetite is triggered by aldosterone & angiotensin. Total amt of Na+ in the body is a primary determinant of ECF (extra cellular fluid) volume. Aldosterone increases Na+ reabsorption & K+ secretion. Angiotension II has additional effects that raise blood pressure, including increased vasopressin secretion, stimulation of thirst, vasoconstriction, and activation of the cardiovascular control center. ANP (atrial natriuretic peptide) and Brain Natriuretic peptide (BNP) enhance Na+ excretion & urinary water loss by increaseing GFR, inhibiting tubular reabsorption of NaCl & inhibiting therelease of renin, aldoterone & vasopressin. Renin converts angiotension in the blood to angiotensin I. ACE converts ANGI to ANG II. Stimuli for release of renin is related (in)directly to low blood pressure.

3-13The term for any hormone that controls the secretion of another hormone is a

TROPHIC HORMONE

6-27Calcium, released by the sarcoplasmic reticulum, binds to ________ to move the ________, which allows actin to form of a ________ with myosin.

TROPONIN C, TROPOMYOSIN, HIGH-FORCE CROSSBRIDGE

10-33________ is a condition of insulin deficiency from beta cell destruction.

TYPE 1 DIABETES MELLITUS

10-39________ is known as insulin-resistant diabetes.

TYPE 2 DIABETES

7M6=Explain the basic components of an ECG and what the P, QRS, and T waves correspond to

The ECG represents the summed electrical activity of all cells recorded from the surface of the body. It records at 25 mm/second. P= atrial depolarization P-R segment= conduction through AV node & AV bundle QRS complex= ventricular depolarization T wave= ventricular repolarization

5M6=Describe anatomical and physiological differences between the sympathetic and parasympathetic divisions

The EFFERENT division of the peripheral nervous system can be subdivided into somatic motor neurons that control skeletal muscles & autonomic neurons that control smooth muscle, cardiac muscle many glands & some adipose tissue. The AUTONOMIC SYSTEM (a.k.a. visceral system, vegetative system) HOUSES THE SYMPATHETIC & PARASYMPATHETIC DIVISIONS. SYMPATHETIC=FIGHT OR FLIGHT, dominant in stressful situations (heart speeds up, blood vessels to muscles of the arms, legs & heart dilate; liver produces glucose to provide energy for muscle contraction; digestion becomes low priority, so blood is diverted from GI tract to skeletal muscles). Mediated through hypothalamus & is a total-body response to a crisis.//the ganglion are close to the spinal cord or along the descending aorta. most originate in the thoracic & lumbar regions of the spinal cord. PARASYMPATHETIC=In command of routine, quiet activities of day-to-day living such as digestion. They are called REST AND DIGEST FUNCTIONS.// the ganglion are located on/near the organ. originate in the brain stem or the sacral region of the spinal cord.

9M2=Describe the tubular elements and vascular features of the nephron

The NEPHRON is the functional unit (smallest structure that can perform all the functions of an organ) of the kidney. Outter=cortex; Inner=medula. The layers are formed by the organized arrangement of microscopic tubules called nephrons. About 80% of nephrons are almost completely contained w/i the cortex, the other 20%, dip down into the medulla. Vascular elements=blood enters thru renal artery before flowing into smaller arteries & then arterioles in the cortex. Blood flows from the afferent arteriole into a ball-like network of capillaries known as the glomerulus. From there into an efferent arteriole, then to the peritublular capillaries that surround the tubule. Then the vasa recta. Renal capillaries join to form venules & small veins, conducting blood out of the kidney through the renal vein. Tubular elements=Begins w/hollow, ball-like structure called Bowman's capsule that surrounds the glomerulus. Both of these are called the renal corpuscle. From there, filtered fluid flows into the proximal tube then to loop of Henle that id divided into 2 limbs (descending limb & ascending limb) w/i the thick segments. The fluid then passes into the distal tubule then to a collecting duct. collecting ducts pass from the cortex thru the medulla & drain into the renal pelvis (now called urine) flows into the ureter on its way to excretion. Loop of Henle loop s7 passes between the afferent & efferent arterioles = juxtaglomerular apparatus.

4M7=Explain the difference between gray and white matter

The cerebrum is responsible for thought and emotion. GRAY=of the CNS is unmyelinated nerve cell bodies, dendrites & axon terminals. The cell bodies either form layers in part of the brain or cluster into groups know as nuclei. WHITE= of the CNS has myelinated axons and run in bundles called tracts. Tight junctions in brain capillaries create a blood-brain barrier that prevents possibly harmful substances in the blood from entering the interstitial fluid.

2Q29There are two forms of diabetes: diabetes mellitus (lack of insulin regulation of cellular uptake of glucose) and diabetes insipidus (lack of regulation of water loss in the urine by the hormone antidiuretic hormone or ADH). In each case, there are two similar explanations for the resulting pathology. What are they? (Hint: Consider general principles of hormone action.)

The endocrine system coordinates functioning between different organs through hormones, which are released into the bloodstream from specific types of cells within endocrine (ductless) glands. Once in circulation, hormones affect function of the target tissue. Some hormones exert an effect on cells of the organ from which they were released (paracrine effect), some even on the same cell type (autocrine effect). Hormones can be peptides of various sizes, steroids (derived from cholesterol), or amino acid derivatives. the hormones are not able to affect the target organ (pancreas) because they have been inhibited or the body/organ outright rejects them. The body either shuts the function down (stops releasing the hormone) or it stops the receptors from receiving them. Whatever has been targeted, has been shut off or shut out.

5-33 3. Provide three reasons for the greater visual acuity at the fovea.

The fovea is the region of most acute vision and the point on which light is focused when you look at an object. The fovea is free of neurons and blood vessels that would block light reception, so photoreceptors receive light directly with minimal scattering. It focuses at the center of your field of vision, so everything is in greater focus at the center coupled with the greatest amount of light. Light has to pass through a couple layers of cells in order to get to the photoreceptors, so light coming in to the eye have to pass through bipolar neurons and ganglion cells. These are the cells that actually receive electrical signals from the photoreceptors and transmit them to the optic nerve to send them to the brain. They are clear cells, but they are somewhat of an obstacle to light passing through them. In the fovea, the bipolar neuron and ganglion cells are going to be bent back so light doesn't have to pass through those cells in order to excite the photoreceptors allowing more light to get through more easily. There are only cones in the fovea. Even though cones require more light to get activated, they create images of higher acuity and they are responsible for all of a person's color vision. There are additional cells called horizontal cells and amacrine cells that can influence the behavior of the bipolar cells and the ganglion cells and control the information that is going to get relayed to the brain. In addition, there is convergence of photoreceptors on to bipolar cells and there's also some convergence on bipolar cells on to ganglion cells. If you have 1 rod cell that will be activated by light, but there are other rod cells that are going to synapse on the same bipolar neuron, that signal from that one rod cell may not be sufficient to cause the bipolar neuron to actually send that information back to the brain to be perceived. There is less convergence in the fovea. There is closer to a 1:1 relationship between the photoreceptors and the bipolar neurons.

10M9=Explain how pancreatic hormones influence cell behavior

The insulin to glucagon ratio regulates metabolism. Insulin is the dominant hormone of the fed state. It is synthesized as an inactive prohormone & activated prior to secretion. Insulin promotes anabolism. The primary targes are the liver, adipose tissue & skeletal muscles. Usual cell response is increased glucose metabolism. sometimes it regulates the GLUT transporters. 1) Insulin increases glucose txport into most, but not all, insulin-sensitive cells. 2) Insulin enhances cellular utilization & storage of glucose. 3) Insulin enhances utilization of amino acids. 4) Insulin promotes fat synthesis. Glucagon is dominant in the fasted state. It is secreted by pancreatic alpha cells, is antagonistic to insulin in its effects on metabolism. The function of glucagon is to prevent hypoglycemia.

9M4=Explain the specific regulation of sodium, water, potassium, and glucose transport in the kidneys

The kidneys keep concentrations of key ions w/i a normal range by balancing dietary intake w/urinary loss. Na+ is the major ion involved int he regulation of extgracellualr fluid volume & osmolarity. K & Ca2+ concentrations are also closely regulated. Glucose, amino acids, ions & various organic metabolites are reabsorbed by Na+ -linked secondary active transport.

3M5=Distinguish between characteristics of the anterior and posterior pituitary glands

The pituitary gland is composed of the anterior pituitary (a true endocrine gland of epithelial origin, derived from embryonic tissue that formed the roof of the mouth) and the posterior pituitary (an extension of the brain). The posterior pituitary stores & releases 2 neurohormones, oxytocin & vasopressin that're made in the hypothalamus. Anterior pituitary secretes 6 hormones (controls growth, metabolism & reproduction): prolactin, growth hormone (somatotropin), follicle-stimulating hormone, lutenizing hormone, thyroid-stimulating hormone & adrenocorticotrophic hormone Pituitary is 2 fused glands.

4M4=Describe how chemical signals are released from cells and act on targets

The vast majority of synapses are chemical. they use neurocrine mole's to carry info from 1 cell to the next. At chemical synapses, the electrical signal of the presynaptic cell is converted into a neurocrine signal that crosses the synaptic cleft and binds to a receptor on its target cell.

4-2 2. Describe the phases of a normal sleepy cycle.

There are 4 stages of the sleep cycle. They are identifiable and predictable. They have characteristic somatic changes and EEG patters. The 2 major phases are Stage 1 (REM—rapid eye movement—sleep) and Stage 4 (slow-wave sleep (also called non-REM sleep or deep sleep). A typical 8-hour cycle consist of repeating cycles between stages 1-4. In the first hour, the person moves from wakefulness to deep sleep; then the sleeper cycles between deep sleep and REM with stages 2-3 between. Near the end of an 8-hour sleep period, a sleeper spends the most time in stage 2 and REM until finally waking for the day. One cycle takes approximately 90 minutes. Before studying the sleep cycles, we must note the types of brain waves during the awake cycle, beta and alpha waves. Beta waves are the highest in frequency, lowest in amplitude and they are not very consistent in their patterns. When relaxed but still awake, our brain waves are alpha—they are synchronous (meditation). Stage 1's (REM sleep) EEG is similar to an awake person. It has low-amplitude, high-frequency waves. During this phase, brain activity inhibits motor neurons to skeletal muscles, paralyzing them—except the eye muscles and breathing. During this phase homeostatic functions are depressed and body temperature falls toward ambient temperature. This is the dreaming phase and the eyes will move around in response to the dream. It is characterized by theta waves that are even slower in frequency and greater in amplitude than alpha waves. Going from relaxation to Stage 1 is gentle and subtle. Stage 2 is theta wave activity interspersed with two wave phenomena called sleep spindles and K complexes that occur every minute or so. Sleep spindles are a sudden increase in wave frequency and K complexes are a sudden increase in wave amplitude. Stages 1 & 2 are light stages of sleep and if the sleeper is awoken during either of these two phases, he will probably think he had not slept at all. Stages 3 and 4 (slow-waves) are recognized on the EEG by delta waves (high-amplitude, low-frequency waves of long duration that sweep across the cerebral cortex). Delta waves are slow and high amplitude waves. There is no real distinction between Stage 3 and Stage 4 other than Stage 3 is when less than 50% of the waves are delta and Stage 4 is when more than 50% of the sleep is delta waves. This is the deepest sleep cycle and when we are most likely to not wake up. It is the most difficult stage to wake a person and if/when they are, they are usually sleepy and disoriented. This is when sleep walking and talking occur as well as when the brain tells the body to adjust its position without conscious commands.

4M10=Explain what happens to a sleeping brain and why sleep is necessary

There are 4 stages of the sleep cycle. They are identifiable and predictable. They have characteristic somatic changes and EEG patters. The 2 major phases are Stage 1 (REM—rapid eye movement—sleep) and Stage 4 (slow-wave sleep (also called non-REM sleep or deep sleep). A typical 8-hour cycle consist of repeating cycles between stages 1-4. In the first hour, the person moves from wakefulness to deep sleep; then the sleeper cycles between deep sleep and REM with stages 2-3 between. Near the end of an 8-hour sleep period, a sleeper spends the most time in stage 2 and REM until finally waking for the day. One cycle takes approximately 90 minutes. Before studying the sleep cycles, we must note the types of brain waves during the awake cycle, beta and alpha waves. Beta waves are the highest in frequency, lowest in amplitude and they are not very consistent in their patterns. When relaxed but still awake, our brain waves are alpha—they are synchronous (meditation). Stage 1's (REM sleep) EEG is similar to an awake person. It has low-amplitude, high-frequency waves. During this phase, brain activity inhibits motor neurons to skeletal muscles, paralyzing them—except the eye muscles and breathing. During this phase homeostatic functions are depressed and body temperature falls toward ambient temperature. This is the dreaming phase and the eyes will move around in response to the dream. It is characterized by theta waves that are even slower in frequency and greater in amplitude than alpha waves. Going from relaxation to Stage 1 is gentle and subtle. Stage 2 is theta wave activity interspersed with two wave phenomena called sleep spindles and K complexes that occur every minute or so. Sleep spindles are a sudden increase in wave frequency and K complexes are a sudden increase in wave amplitude. Stages 1 & 2 are light stages of sleep and if the sleeper is awoken during either of these two phases, he will probably think he had not slept at all. Stages 3 and 4 (slow-waves) are recognized on the EEG by delta waves (high-amplitude, low-frequency waves of long duration that sweep across the cerebral cortex). Delta waves are slow and high amplitude waves. There is no real distinction between Stage 3 and Stage 4 other than Stage 3 is when less than 50% of the waves are delta and Stage 4 is when more than 50% of the sleep is delta waves. This is the deepest sleep cycle and when we are most likely to not wake up. It is the most difficult stage to wake a person and if/when they are, they are usually sleepy and disoriented. This is when sleep walking and talking occur as well as when the brain tells the body to adjust its position without conscious commands.

3-10 1. Dexamethasone is a drug used to suppress the secretion of ACTH from the anterior pituitary gland. Two patients with hypersecretion of cortisol are given dexamethasone. Anika's cortisol secretion level falls to normal after the medication, but Bonnie's cortisol secretion remains elevated. Which patient has primary hypercortisolism, also known as Cushing's syndrome? Explain your reasoning.

There is a difference in Cushing's disease and Cushing's syndrome. Cushing's DISEASE is associated with the pituitary gland and DOES HAVE an effect on ACTH. The pituitary problem would be an issue with secondary hypersecretion. Therefore, Anika would have been affected by Cushing's DISEASE (Silverthorn, 2013, p. 229). Cushing's SYNDROME is from hypercortisolism caused from any cause (like iatrogenic -physician caused—or adrenal issues, as examples) other than pituitary (Silverthorne, 2013, p. 781). Cushing's SYNDROME (also known as primary hypercortisolism) would have no effect on ACTH and the dexamethasone would have no effect on the cortisol. Since this is Bonnie's case, then she would have the Cushing's syndrome probably caused from the adrenals not functioning properly or a tumor.

6M2=Describe the unique features of muscle cells

They generate motion, force & heat.

8-31Timmy is a cantankerous toddler who has just threatened that he will hold his breath until Mom gives him some chocolate. His mother refuses to be manipulated and watches in amusement as Timmy stubbornly refuses to breathe. To her horror, Timmy loses consciousness and collapses onto the floor. Her cousin, who is enrolled in a course for emergency medical technical (EMT) training, is visiting, and tells her there is no need to call for an ambulance. Why did Timmy lose consciousness? Should his mother trust her instincts and call for help and begin CPR, or should she listen to her well-meaning but young and inexperienced cousin? Explain.

Timmy is a cantankerous toddler who has just threatened that he will hold his breath until Mom gives him some chocolate. His mother refuses to be manipulated and watches in amusement as Timmy stubbornly refuses to breathe. To her horror, Timmy loses consciousness and collapses onto the floor. Her cousin, who is enrolled in a course for emergency medical technical (EMT) training, is visiting, and tells her there is no need to call for an ambulance. Why did Timmy lose consciousness? Should his mother trust her instincts and call for help and begin CPR, or should she listen to her well-meaning but young and inexperienced cousin? Explain. Tommy lost consciousness because of a lack of oxygen to the brain. Chemoreceptor reflexes make it impossible for children to carry out that threat. Higher brain centers affect patterns of ventilation. His mother should not to call an ambulance and start CPR (assuming that the spell only lasts a moment), she should listen to her cousin, the EMT. The mother should make an appointment with her health care provider to double check that there are no underlying health issues, but Timmy should be ok. Breath holding spells are a behavior problem coupled with an abnormal reflex (only about 5% of children hold their breath until the point of passing out) with the child and in most cases the child will regain consciousness again in just a few seconds. It is normally triggered by frustration, anger, fear or other strong emotional reactions. This can happen to children between the ages of 6 months to 6 years of age and the child will usually outgrow this bad behavior. Some experts believe that the children are able to hold their breath until they pass out due to an iron deficiency or anemia, so an iron supplement may help this phase pass.

8M2=Describe the anatomy of the respiratory system

UPPER= mouth, nasal cavity, pharynix & larynx. (Filters, warms & humidifies inhaled air) LOWER= trachea, bronchi, bronchioles & exchange surfaces of the alveoli.

9-30Urine is carried out of the body by the ________.

URETHRA

1M6=Explain why water is essential for life as we know it

Universal solvent=water. Free H+ can disrupt noncovalent bonds & alter it's ability to function. The pH measures Hydrogen ion concentration. Buffers moderated pH changes.

9-32The ________ are the long peritubular capillaries that dip into the medulla.

VASA RECTA

7-32An increase in blood vessel diameter is known as ________.

VASODILATION

6-26Autonomic reflexes are also called ______________ reflexes.

VISCERAL

8M9=Describe the ways that carbon dioxide is transported in the blood

Venous blood carries 7% of its CO2 dissolved in plasma, 23% as carbaminohemoglobin & 70% as bicarbonate ion in the plasma. Carbonic anhydrase in RBCs converts CO2 to carbonic acid, which dissociates into H+ and HCO3-. The H+ binds to hemoglobin & the HCO3- enters the plasma using the chloride shift.

4M6=Describe the organization and integration of neural pathways and how multiple neurons can synapse on a single target cell

When a presynaptic neuron synapses on a larger # of postynaptic neurons, the pattern is known as divergence. When several presynaptic neurons provide input to a smaller # of postynaptic nurons = convergence. The summation of simultaneous graded potentials from different neurons = spatial summation.

2M1=describe the function of enzymes and the role of enzymes in cell metabolism

biological catalysts that speed up chem reactions w/o being changed themselves. The reactants catalyzed by enzymes=substrates. enzymes that must be activated require a COFACTOR. Organic cofactors=COENZYMES. Activity is altered by temp, pH & modulator molecules. They work by lowering the activation energy of a reaction. Energy is the capacity to do work.

6-19The T-tubular membrane contains ________ receptors that are ________ and therefore respond to action potentials. This receptor is mechanically linked to ________ receptors in the adjacent sarcoplasmic reticulum.

dihydropyridine (DHP) or L-type calcium channel, voltage-sensing, ryanodine (RyR) or calcium release channel

1M1=Define homeostasis and relate it to the study of physiology

maint of relatively constant internal enviro. Variables: pH, temperature, ion concentrations, oxygen & water. Failure = disease. internal environment is called extracellular fluid. whole body has adapted w/change, but most cells can tolerate much less change. law of mass balance = amt of substance in body is to remain constant, any input must be offset by an equal loss. input into body comes from metabolism or from outside enviro. output occurs through metabolism or excretion. cells & extracellular fluid both maintain homeostasis, but they are not identical in composition. Their stable condition is a dynamic (moves back & forth) steady state (no net movement). most solutes are concentrated in either 1 compartment or the other, creating a state of disequilibrium. Regulated variables have a set-point and a normal range. The simplest homeostatic control takes place at the tissue or cell=local control. Local control restricted to a tissue/cell involved. Control systems have 3 components: input signal, integrating center & output signal. Reflex pathways are response loops (begins when a stimulus is sensed by a sensor) & feedback loops (moderate the response loop). Sensor is linked by input signal to integrating sensor that decides on appropriate response. Output signal travels form integrating ctr to target that carries out appropriate response. Negative feedback loop=(homeostatic) opposes or removes the original stimulus & stops the loop. + Feedback loop reinforces stimulus & destabilizes the stm until intervention outside loop stops response. Feedfwd control=allows body to predict upcoming change & starts the response loop in anticipation of change.

10-29Match the structure to its function. You may use the structures more than once. A. rectum B. small intestine C. stomach D. mouth E. large intestine 1. Carbohydrate digestion begins here 2. Fat digestion begins here. 3. Protein digestion is completed here. 4. Fat digestion is completed here. 5. Carbohydrate digestion is completed here. 6. Protein digestion begins here.

mouth:Carbohydrate digestion begins here, small intestine:Carbohydrate digestion is completed here., stomach:Protein digestion begins here., small intestine:Protein digestion is completed here., stomach:Fat digestion begins here., small intestine:Fat digestion is completed here.


Ensembles d'études connexes

Cap. 8: Gente que come bien DESCRIPCIONES DE COMIDA

View Set

Chapter 16: Disorders of the Immune Response Prep U

View Set

Med Surg Test 5- Review Questions- Chapter 49

View Set

mechanical properties of materials

View Set

NSG 333 Ch 11- Maternal Adaptation During Pregnancy

View Set